Schwarze Löcher gehören zu den missverstandensten Himmelskörpern überhaupt. Viele stellen sie sich als eine Art Staubsauger vor, der alles gnadenlos ansaugt und vor dem es kein entkommen gibt. Das ist falsch, wie ich in diesem Artikel ausführlich erklärt habe.Schwarze Löcher sind extrem kompakte Überreste von ehemaligen Sternen. Abgesehen von den supermassereichen schwarzen Löchern in den Zentren von Galaxien sind die stellaren Löcher auch nicht wahnsinnig viel schwerer als normale Sterne. Da sie aus Sternen entstehen geht das auch gar nicht anders. Das besondere an ihnen ist nicht ihre Masse, sondern ihre Dichte. Masse krümmt die Raumzeit. Wenn immer mehr Masse auf immer kleineren Raum konzentriert wird, wird die Raumkrümmung immer stärker und stärker, bis man schließlich keine normale “Delle” mehr in der Raumzeit hat, sondern quasi ein “Loch”. Alles was eine gewisse Grenze – den “Ereignishorizont” – überschreitet, kann aus dieser Region extrem gekrümmter Raumzeit nicht mehr entkommen. Außerhalb davon besteht allerdings keine Gefahr. Das schwarze Loch “saugt” einen nicht einfach an… Ein Planet könnte problemlos ein schwarzes Loch umkreisen, ohne hinzufallen, vorausgesetzt er hält den nötigen Sicherheitsabstand ein.


Vor diesen schwarzen Löchern müssen wir keine Angst haben. Sie finden sich zwar überall in den Galaxien, aber es besteht keine Gefahr, dass eines davon plötzlich hinterrücks die Erde anspringt und uns Schaden zufügt. Auch wenn die Löcher schwarz sind und wir sie nicht direkt sehen können, sind sie doch auch schwer. Schwerer als unsere Sonne – und so ein massereiches Objekt kann uns nicht überraschen. Sollte tatsächlich einmal eines ausreichend nahe kommen um eine Gefahr darzustellen – was wahnsinnig unwahrscheinlich ist; in der Milchstraße ist so viel Platz, dass nahe Begegnungen dieser Art so gut wie nie vorkommen – dann wüssten wir das schon Jahrzehnte vorher, weil wir die Auswirkungen der Gravitationskraft des schwarzen Loches sehen könnten. Es ist so schwer wie ein großer Stern und wir würden lange vorher merken, dass sich die äußeren Planeten und Asteroiden unseres Sonnensystems nicht mehr so bewegen, wie sie es tun sollten, weil ihre Bahn vom nahenden schwarzen Loch gestört werden.

i-dfa9ae6bfd99bc9f849c9ba21bd35c92-BH_LMC-thumb-500x400.png
Simulierte Ansicht eines schwarzen Lochs, das sich zwischen uns und der großen Magellanschen Wolke befindet (Bild: CC-BY-SA 2.5)

Um die stellaren schwarzen Löcher müssen wir uns also keine Sorgen machen. Neben den großen stellaren schwarzen Löchern, die aus Sternen entstehen, gibt es aber vielleicht auch noch eine andere Art. Winzig kleine, sogenannte “primordiale” schwarze Löcher. Sie sind so klein wie Elementarteilchen, wiegen vielleicht so viel wie ein durchschnittlicher Asteroid und entstanden möglicherweise direkt nach dem Urknall. Sie zu entdecken ist natürlich viel schwerer. Sie sind, ebenso wie ihre großen Kollegen, schwarz und nicht zu sehen. Und sie sind leicht genug, damit ihre gravitativen Störungen nicht weiter auffallen. Es kann durchaus sein, dass sie sich der Erde unbemerkt nähern können. Ist das ein Grund zur Besorgnis?

Nein, eigentlich nicht. Nochmal: schwarze Löcher sind keine Staubsauger. Gefährlich wird es erst hinter dem Ereignishorizont. Der liegt um so näher am schwarzen Loch, je kleiner bzw. leichter es ist. Einem primordialen schwarzen Loch müsste man schon sehr nahe kommen, damit irgendwas passiert. Der Ereignishorizont ist hier nur ein paar Nanometer vom schwarzen Loch entfernt. Würde so ein kleines schwarzes Loch also auf die Erde treffen, wäre das nicht katastrophal. Die Erde würde nicht plötzlich verschluckt oder aus der Bahn geworfen werden. Ganz folgenlos würde die Kollision aber trotzdem nicht bleiben. Und das ist gut so, denn dadurch haben wir zumindest theoretisch die Möglichkeit, diese primordialen schwarzen Löcher zu entdecken!

Yang Luo und sein Kollegen von der Universität Princeton bzw. dem Max-Planck-Institut für Sonnensystemforschung in Katlenburg-Lindau haben genau untersucht, was im Falle einer Kollision zwischen Erde und schwarzem Loch passieren würde. Was auch immer passiert, es dauert auf jeden Fall nicht lange. So ein primordiales schwarzes Loch bewegt sich mit 200 bis 400 Kilometer pro Sekunde. Es wäre also in knapp einer Minute durch die Erde hindurch gerauscht. Was es in dieser Zeit anstellt, haben Luo & Co in detallierten Computersimulationen untersucht. Auf seinem Weg durch die Erde regt das schwarze Loch seismische Wellen an. Es würden Erdbeben entstehen, allerdings keine sehr starken. Ein typisches primordiales schwarzes Loch mit einer Masse von 1015 Gramm erzeugt höchstens ein Beben der Stärke 4.

Hier sieht man ein Bild der in der Erde angeregten Erdbebenwellen. Das schwarze Loch dringt oben in die Erde ein und tritt unten wieder aus. Unterwegs regt es sogenannte “zylindrische Wellen” an. An den verschiedenen Übergangsschichten – zum Beispiel zwischen Erdkruste und Erdmantel oder zwischen innerem und äußerem Kern – entstehen aber auch zusätzlich “sphärische” Wellen, die dann überall auf der Erdoberfläche gleichzeitig eintreffen.

i-0f44b6d9e1b51a4ad7b97255ada17ed9-blackhole-thumb-500x462.png

So etwas kann bei normalen Erdbeben nicht entstehen und würden die Seismometer der Geologen Wellen dieser Art registrieren, wäre das ein starker Hinweis auf die Existenz primordialer schwarzer Löcher. Eine Entdeckung dieser Art wäre revolutionär. Nicht nur, dass wir dann die Vorgänge kurz nach dem Urknall besser verstehen würden, wir hätten vielleicht auch einen Bestandteil der dunklen Materie identifiziert. Wenn es primordiale schwarze Löcher gibt, dann könnten sie einen Teil der dunklen Materie ausmachen und Luo et al. schreiben in ihrem Artikel, dass prinzipiell auch die gesamte dunkle Materie unserer Milchstraße aus primordialen schwarzen Löcher bestehen könnte.

Leider ist aber die Chance sehr gering, dass wir die kleinen schwarzen Löcher tatsächlich auf diese Art und Weise finden werden. Eine Kollision zwischen ihnen und der Erde ist nur alle paar Millionen Jahre zu erwarten. Aber wir sollten trotzdem die Augen offen und die Seismometer im Blick behalten. Wer weiß, vielleicht haben wir ja Glück!

Kommentare (209)

  1. #1 JK
    21. März 2012

    “Da sie aus Sternen entstehen geht das auch gar nicht anders” – muss es vermutlich heißen.

    Eine Kollision zwischen ihnen und der Erde ist nur alle paar Millionen Jahre zu erwarten.

    Das bedeutet, solche Kollisionen gab es schon oft? Und warum fliegt das schwarze Loch überhaupt durch die Erde hindurch, warum macht es nicht “Krawumm”? Und warum kann so wenig Materie so dicht zusammengepresst werden, dass es ein kleines schwarzes Loch gibt, das dauerhaft hält und sich nicht auflöst (was immer das bedeuten mag)?

  2. #2 Dark_Tigger
    21. März 2012

    Ganz dumme Frage, warum genau verhalten sich derartige Schwarze Löcher nicht genauso, wie Meteoriten, gleicher Masse und Geschwindigkeit?
    (Also es macht kräftig Rums und gut ist?)

  3. #3 Florian Freistetter
    21. März 2012

    @JK, Dark Tigger: “Und warum fliegt das schwarze Loch überhaupt durch die Erde hindurch, warum macht es nicht “Krawumm”?”

    Naja, warum geht eine Pistolenkugel durch eine Melone hindurch und macht nicht Krawumm? 😉 Das primordiale schwarze Loch ist so groß wie ein Atom. Und so schwer wie ein Asteroid. Aus seiner Sicht ist die Erde eine flauschige Kugel die keinen Widerstand leistet. Das geht glatt durch.

    “Und warum kann so wenig Materie so dicht zusammengepresst werden, dass es ein kleines schwarzes Loch gibt, das dauerhaft hält und sich nicht auflöst (was immer das bedeuten mag)? “

    Die Bedingungen für die Entstehung solcher Löcher gabs nur beim Urknall; da haben die Energien gereicht um Masse so zu komprimieren. Und auflösen (dank Hawkingstrahlung) tun die sich auch – leben aber lang genug, um heute noch da zu sein.

  4. #4 Joachim Pense
    21. März 2012

    Wieso kann man überhaupt von einer Größe eines schwarzen Loches sprechen, die den Ereignishorizont unterschreitet? Da von innerhalb des Ereignishorizontes keine Informationen nach außen dringen können, dachte ich, es wäre auch nicht möglich, irgendwelche Aussagen über die Verhältnisse innerhalb dieses Horizontes zu machen. Das schwarze Loch würde sich mithin auf den gesamten Bereich des Ereignishorizontes erstrecken.
    Joachim

  5. #5 Florian Freistetter
    21. März 2012

    @Joachim Pense: “Wieso kann man überhaupt von einer Größe eines schwarzen Loches sprechen, die den Ereignishorizont unterschreitet?”

    Warum soll man es nicht können? Der Ereignishorizont und das schwarze Loch sind unterschiedliche Dinge.

  6. #6 elias
    21. März 2012

    @joachim
    Normalerweiße meint man mit der Größe eines schwarzen Lochs die Ausdehnung des Ereignishorizonts.

  7. #7 Brucer
    21. März 2012

    Eine Frage zu Schwarzen Löchern, die mich schon länger beschäftigt (und zu der ich auch nach langem googeln nichts gefunden habe):

    Unsere Galaxie dreht sich doch um ein Schwarzes Loch. Ich weiss, dass man das natürlich nicht sehen kann, aber woran kann ich mich am Nachhimmel orientieren, um in die richtige Richtung zu schauen?

    (Entschuldige, Florian, dass ich diesen Schwarzloch-Artikel schamlos für meine Frage genutzt habe!)

  8. #8 Jaque de Roek
    21. März 2012

    Wie sieht das aber mit vagabundierenden (normalen) schwarzen Löchern aus?
    Es gibt ja Planeten und Sonnen, die aus ihrem System herausgeschleudert werden und nun quasi frei durch die Raumzeit fliegen. Ok, man koennte sie einigermaßen früh entdecken, und die Wahrscheinlichkeit, dass sie (oder ihr Ereignishorizont) auf Kollisionskurs mit der Erde ist, mag seeehr gering sein, aber zuende wäre es mit uns doch trotzdem. Die Zeitspanne zwischen Entdeckung des Lochs und Konsum der Erde würde jedenfalls nicht ausreichen, um “auszuwandern”.
    Ok, wieder ein Ereignis mit vernachlässigbarer Wahrscheinlichkeit, aber wie das mit dem Gesetz der großen Zahlen ist, wissen wir ja.

  9. #9 Waaahsabi
    21. März 2012

    Was das Verhalten von Licht innerhalb des EH angeht: Kann man sich das in etwa so vorstellen, daß die Lichtwellen aufgrund des gekrümmten Raumes auch immer um den zentralen Körper kreisen?

  10. #10 Bullet
    21. März 2012

    @Brucer:

    Unsere Galaxie dreht sich doch um ein Schwarzes Loch. Ich weiss, dass man das natürlich nicht sehen kann, aber woran kann ich mich am Nachhimmel orientieren, um in die richtige Richtung zu schauen?

    Wenn dieses Schwarze Loch strahlend hell wäre, könntest du es trotzdem nicht sehen. Denn in Richtung des galaktischen Zentrums gibt es haufenweise Staub, der noch viel mehr verdunkelt als nur so ein popeliges Fünzelchen von 10 000 Sonnenleuchtkräften.
    anyway: die Richtung zum galaktischen Zentrum ist bekannt. Es liegt im Sternbild Schütze etwas “links unterhalb” des Teekesselchen-Asterismus, der den auffälligsten Teil des Sternbildes ausmacht. Dummerweise ist der von Deutschland aus quasi nicht zu sehen. Da solltest du schon im Sommer nach Südspanien fahren – da könntest du mehr Glück haben.

  11. #11 Kallewirsch
    21. März 2012

    aber zuende wäre es mit uns doch trotzdem.

    Warum sollte es das?

    Nochmal: die Annahme war ein primordiales SL. Das ist so winzig, dass aus seiner Sicht die Erde wie ein grobmaschiges Sieb aussieht, bei dem an und zu mal irgendwo ein Atom rumlungert. Das saust da durch, wie eine Bowlingkugel durch einen Wald von Bowlingpins, die 100 Meter weit auseinanderstehen.
    Ab und zu trifft das SL natürlich auf einen Pin, äh Atom. Das verleibt es sich ein und hat dann eine Atommasse mehr. Darauf kommts bei etwas mehr als einer Sonnenmasse total auch nicht wirklich an. Deswegen wächste das SL nicht wesentlich. Und für die Erde spielt das auch keine wirkliche Rolle, ob man da jetzt ein paar Atome aus dem Inneren ‘rauskegelt’ oder nicht.

    Der Fehler, der immer wieder gemacht wird, besteht darin, dass von der Vorstellung ausgegangen wird, dass die ‘Massivität’ die wir im Alltag wahrnehmen sich bis zum Allerkleinsten fortsetzt. Ein Stück Wurst ist massiv und hat keine Löcher. Geh ich aber in der Vergrößerung runter bis auf atomare Ebene bzw. noch kleiner, dann stellt man fest: Hauptsächlich ist da eines; nämlich viel, viel leerer Raum zwischen winzigen Elementarteilchen. Das du auf dem Boden stehen kannst und nicht durch die Atome durchfällst, beruht NICHT darauf, dass Atome massive Kugeln sind die bei Kontakt aneinander anstehen, sondern dass sie sich elektrostatisch abstossen. Das verhindert, dass sich Festkörper durchdringen können, wie 2 Wolken Schrotkugeln mehr oder weniger ungehindert im 90° Winkel sich durchdringen ohne das besonders viele Kugeln zusammenstossen.

  12. #12 Mathias
    21. März 2012

    Nimmt das Schwarze Loch bei der Kollision mit der Erde Materie auf? Also wird es schwerer? Kann es mit der Zeit auch größer werden bis zu einem “normalen” Schwarzen Loch wenn es sagen wir mal um das Zentrum der Sonne kreisen würde (aber innerhalb ihres Radius)?

  13. #13 Florian Freistetter
    21. März 2012

    @Mathias: “Nimmt das Schwarze Loch bei der Kollision mit der Erde Materie auf? Also wird es schwerer?”

    Nicht wesentlich, würde ich sagen. Es ist ja nicht mal ne Minute innerhalb der Erde und so klein wie Atom. Da müsste die andere Materie dem SL schon auf Atomdistanz nahekommen, damit es ein bisschen was fressen kann. Vielleicht hat das SL nach der Passage ein paar Atome der Erde verschluckt…

    “Kann es mit der Zeit auch größer werden bis zu einem “normalen” Schwarzen Loch wenn es sagen wir mal um das Zentrum der Sonne kreisen würde (aber innerhalb ihres Radius)? “

    Hmm – das würde dann vermutlich SEHR lange dauern. Da stellare SL immer schwerer sind als die Sonne müsste es dabei auch die Sonne komplett auffressen.

  14. #14 troglodyt
    21. März 2012

    @Jaque de Roek:
    Wir gehen wahrscheinlich schon unter bzw. drauf wenn ein SL der normalen Größenordnung auch nur in die Nähe (in astronomischen Einheiten) der Erde kommt. Die Gravitationskraft würde die Erdbahn um die Sonne massiv beeinflussen und für das was dann passiert ist Klimawandel ein Euphemismus.

  15. #15 elias
    21. März 2012

    @troglodyt, Jaque de Roek:
    Die selben Effekte hätten wir, wenn jeder x-beliebige Stern so nahe an unser Sonnensystem kommt. Die Planetenbahnen können instabil werden und die Erde in die Sonne stürzen. Ob das jetzt durch ein schwarzes Loch oder einen anderen Stern verursacht wird ist ja egal, die Gravitationswirkung eines SL ist nicht auf magische weiße größer als die eines Sterns.

  16. #16 Alderamin
    21. März 2012

    @Kallewirsch

    Jaque de Roek sprach von vagabundierenden Schwarzen Löchern und nannte als Beispiel vagabundierende Planeten und Sonnen (im Prinzip vagabundieren alle Sonnen irgendwie durch die Milchstraße, unsere auch, die folgen keinem strengen Kepler-Orbit). Also meint er wohl ein Stellares Schwarzes Loch.

    Nun, hier gilt folgendes: Stellare Schwarze Löcher entstehen nur aus Riesensternen mit mindestens 10 Sonnenmassen. Die sind aber gemessen an der Gesamtpopulation der Sterne eher selten. Die Chance, dass ein solches Schwarzes Loch das Sonnensystem durcheinander bringt, ist daher klein, aber nicht 0, und wenn das einträte, dann wäre das ungesund für uns (Asteroideneinschläge), im Extremfall sehr ungesund (Änderung der Umlaufbahn bis zum Herauskatapultieren aus dem Sonnensystem)

    Wenig gesünder wäre im übrigen die enge Begegnung mit einem normalen Stern, von denen es 20- oder 50-mal mehr gibt. Dass unser Sonnensystem nach 4,5 Milliarden Jahren noch immer friedlich kreist, zeigt aber, dass so enge Begegnungen, dass es der Erde (oder auch nur Jupiter) an den Hals geht, sehr selten sind. Die Abstände zwischen den Sternen sind hundertausende Male größer als die zwischen den Planeten und der Sonne. Man mag sich an dem Restrisiko ergruseln, aber die Chance, von einem Hai gefressen zu werden, ist deutlich größer. Ganz zu schweigen vom Tod im Straßenverkehr.

  17. #17 Ucuri
    21. März 2012

    Interessanter Artikel!

    Aber rein aus Interesse mal ein ganz abwegiges Szenario:
    Wenn so ein primordiales schwarzes Loch direkt durch einen Menschen ‘hindurchrast’, würde er etwas davon merken?
    Also würde es quasi ‘seismische’ Wellen im Körper geben? ^^

  18. #18 Florian Freistetter
    21. März 2012

    @Ucuri: “Wenn so ein primordiales schwarzes Loch direkt durch einen Menschen ‘hindurchrast’, würde er etwas davon merken? “

    Gute Frage. Mit 400 km/s ist es enorm schnell durch. Ob da Zeit für die Gravitation bleibt, etwas anzurichten? Recht gesund wird es vermutlich aber nicht sein…

  19. #19 rolak
    21. März 2012

    Das ist die seit langem bevorzugte Erklärung für ‘plötzliches Unwohlsein’, Florian 😉

    ..oder für kontextlosen Schluckauf.

  20. #20 JK
    21. März 2012

    @ rolak:

    Das ist die seit langem bevorzugte Erklärung für ‘plötzliches Unwohlsein’

    … und für die gelegentlichen Blackouts bei Politikern, denen wurde das Erinnerungsatom weggefegt.

    @ Florian: Gibt es empirische Hinweise, dass solche “primordialen schwarzen Löcher” überhaupt (in größerer Zahl?) existieren oder hat man die eher als theoretische Möglichkeit zu sehen?

  21. #21 rolak
    21. März 2012

    Tja, so etwas ist schon bitter für Monosynapten, JK
    btt: A primordial black hole is a hypothetical type of black hole, d.h. ein PBH würde zum aktuellen Kosmologiekonsens passen, wäre auch prinzipiell nachweisbar, ist allerdings bisher nicht geschehen.

  22. #22 Florian Freistetter
    21. März 2012

    @JK: Ne, die primordialen schwarzen Löcher sind bis jetzt rein hypothetisch.

  23. #23 Tatjana
    21. März 2012

    Was würde denn bei einer Kollission zwischen einem großen schwarzen Loch und der Erde passieren???

  24. #24 Florian Freistetter
    21. März 2012

    @Tatjana: “Was würde denn bei einer Kollission zwischen einem großen schwarzen Loch und der Erde passieren??? “

    Da würde die Erde natürlich zerstört werden. ABER DAS PASSIERT NICHT! (ich hab das ja extra lang und breit in der Einleitung erklärt).

  25. #25 Nedo
    21. März 2012

    interessant, diese “primordialen” schwarze Löcher haben aber nichts gemeinsam mit denen
    die im HLC beim zusammenprall von Teilchen “enstehen können”? Diese müssten ja nochmals
    schwächer (kleiner) sein, oder?

  26. #26 Florian Freistetter
    21. März 2012

    @Nedo: Richtig, die primordialen SL sind was anderes als die mikro-schwarzen Löcher vom LHC. Die sind so leicht, dass sie sofort wieder zerfallen; die primordialen halten ein paar Milliarden durch weil sie viel massiver sind.

  27. #27 Plastefisch
    21. März 2012

    https://sciencev1.orf.at/science/news/51369

    Offenbar hat man Erdbebenwellen solcher
    Art schon gemessen, auch die Staerke von 4,5
    passt recht gut.

  28. #28 Florian Freistetter
    21. März 2012

    @Plastefisch: Ok, aber Strangelets sind wieder was anderes; die sind keine Primordialen SLs.

  29. #29 Theres
    21. März 2012

    @Florian F
    Sicher? Die Beschreibungen und auch die Erdbebenwellen, auch das Verantwortlichsein für die dunkle Materie ähneln sich aber auffallend.

  30. #30 Theres
    21. März 2012

    Sind doch beide hypothetisch, also, es könnte sogar sein, dass eines von beiden zweimal gesichtet wurde – laut dem Link von Plastefisch.

  31. #31 Plastefisch
    21. März 2012

    @Florian Freistetter
    Wie wuerde das SL eigentlich auf den Erdkoerper einwirken,
    nur durch Gravitation? Ich fand halt die Aehnlichkeit der
    beobachteten Beben, mit den hier genannten Wirkungen
    interessant, und frage mich wie Unterscheidbar die Beben
    der durchschlagenden Partikel waehren.

  32. #32 Tatjana
    21. März 2012

    @ Florian

    Vielen Dank! Ja, ich habe deinen Artikel ja auch gelesen. Wusste nur nicht, welche Gefahr durch ein großes schwarzes Loch droht!

  33. #33 Wurgl
    21. März 2012

    Kurz noch zu diesen Strangelets. Bei dem Affenzahn den die draufhaben, im Artikel steht was von 1,5 Millionen km/h (ca. 415 km/sec), scheinen die nicht mehr unbedingt an die Galaxis gebunden zu sein. Oder lieg ich da falsch?

  34. #34 Peer Schaefer
    21. März 2012

    @Florian: Lieber Florian, ich fand die Frage von Joachim Pense (21.03.12 · 16:29 Uhr) völlig berechtigt. Deine kurze Antwort hat das Problem nicht ausgeräumt. Macht es Sinn, über einen Gegenstand Aussagen zu treffen, wenn dieser Gegenstand PRINZIPIELL unbeobachtbar ist? Ich vermute, dass es auf die Antwort von Joachim Pense eine Antwort gibt (evtl. hat es etwas mit dem Hawking-Zerfall Schwarzer Löcher zu tun: irgendwann kommt doch alles wieder zum Vorschein, d.h. das Innere ist nicht völlig unbeobachtbar – jedenfalls nicht für ewig). Aber die Antwort scheint nicht trivial zu sein.

  35. #35 Alderamin
    22. März 2012

    @Peter

    Aus den Gleichungen der allgemeinen Relativitätstheorie ergibt sich für das Zentrum eines Schwarzen Lochs eine punkt- oder ringförmige Singularität, die von einem Ereignishorizont umgeben ist, jenseits dessen Zeit und Raum gewissermaßen ihre Rollen tauschen. Ob die Singularität in Wahrheit nicht nur 0- bzw. eindimensional ist, sondern wie von der Stringtheorie vorgeschlagen eine winzige Mindestausdehnung in 3 Dimensionen haben muss, ist ein Detail. Gewisse Aussagen über das Innere eines SLs kann man treffen. Z.B. über den Durchmesser der Ringsingularität.

  36. #36 Mathias
    22. März 2012

    @Florian: Danke für die Antowrt. Dann frage ich mich aber, wie ein primordiales SL ein Erdbeben auslösen kann, wenn es nur mit max. ein paar Atomen kollidiert und auch die Gravitationskraft nicht größer als die eines Asteroiden ist?

  37. #37 Ratiomania
    22. März 2012

    @ Florian nochmal zu “Wirkung auf den Menschen”:

    Das Teil ist ca. 5mikrosekündlein im Menschen (wenn direkt von oben kommend) kann man nicht einfach die Berechnungen des von dir zitierten Papers nehmen und auf kleinere Massen/Variablen umrechnen (und vereinfachen)?

    Klar ist dasne Frage der Kategorie “was passiert wenn ich meine Hand in den LHC halte?”:

  38. #38 adenosine
    22. März 2012

    Die Tatsache, dass es in Mathe-Gleichungen Singularitäten gibt, beweist noch lange nicht, dass diese auch physikalisch existieren. Was sind den zur Zeit die besten Indizien, dass Ereignishorizont und Singularität tatsächlich existieren?

  39. #39 Hives
    22. März 2012

    Immer wieder interessant die SL-Artikel zu lesen ^.^ Die sind in der Tat was cooles, was mysteriöses möchte man fast sagen 😉

  40. #40 noch'n Flo
    22. März 2012

    @ rolak:

    Das ist die seit langem bevorzugte Erklärung für ‘plötzliches Unwohlsein’, Florian 😉

    Hey, das wäre doch mal ‘ne neue Ausrede, wenn man keinen Bock hat zur Arbeit zu gehen – “Chef, ich kann heute nicht kommen, letzte Nacht ist ein primordiales SL durch mich hindurchgerast, jetzt leide ich immer noch unter den Gravitationswellen.” Ob man damit wohl durchkommt?

    Scheisse, warum bin ich bloss selbstständig – ich würde das doch zu gerne mal ausprobieren…

  41. #41 Florian Freistetter
    22. März 2012

    @Plastefisch: “Wie wuerde das SL eigentlich auf den Erdkoerper einwirken, nur durch Gravitation?”

    Gar nicht. Wie gesagt, das Teil ist so schwer wie ein Asteroid. Da merkt die Erde nicht viel davon.

  42. #42 Florian Freistetter
    22. März 2012

    @Peer Schaefer: “Macht es Sinn, über einen Gegenstand Aussagen zu treffen, wenn dieser Gegenstand PRINZIPIELL unbeobachtbar ist?”

    Ich habe auch keine Aussagen getroffen. Ich habe nur gesagt, dass es schwarze Löcher gibt, die einen Ereignishorizont haben und so lange nichts passiert, bis man diesen Horizont überschreitet. Wo liegt das Problem?

  43. #43 Alderamin
    22. März 2012

    @adenosine

    Was sind den zur Zeit die besten Indizien, dass Ereignishorizont und Singularität tatsächlich existieren?

    Dass Schwarze Löcher mit einem Ereignishorizont existieren kann man daran verifizieren, dass es massive, dunkle Objekte mit minimaler Ausdehnung gibt. So lassen sich die Supermasssiven Schwarzen Löcher in anderen Galaxien daran nachweisen, dass man sie anhand sie umlaufenden Sternen “wiegen” kann. Die Tatsache, dass von einem millionen Sonnenmassen schweren Objekt nichts zu sehen ist, ist ein Indiz für den Ereignishorizont. Ebenso kann man im Falle von Akkretion Effekte von Rotverschiebung beobachten, die auf den schnellen Umlauf der Materie rückschließen lassen. Siehe dazu https://www.wissenschaft-online.de/astrowissen/lexdt_s02.html#sl, “Wie man ein Schwarzes Loch entdeckt”. Im Augenblick wird gerade ein Projekt vorbereitet, bei dem man mit Radioteleskop-Interferometrie den Ereignishorizont des Schwarzen Lochs im Zentrum der Milchstraße auflösen will. Wir werden ihn dann “sehen” können, wenn auch nur im Radiobereich.

    Was die Singularitäten betrifft: für ihre Existenz gibt es keinen Beleg, und ob sie tatsächlich existieren, ist eher fragwürdig, sie sind eher ein Hinweis darauf, wo die Relativitätstheorie an ihre Grenze stößt. Im oben verlinkten Artikel steht weiter unten etwas zur Singularitätenfrage, und in der Tat gibt es wohl neuerdings einige Modelle, die nicht nur aus Punkt- und Ringsingularitäten kleine ausgedehnte Objekte machen, sondern die das Schwarze Loch innen komplett anders beschreiben.

    Man muss dabei aber auch im Hinterkopf behalten, dass die Allgemeine Relatitvitätstheorie die einzige unter diesen ist, die vielfach experimentell bestätigt wurde. Dass ein anderes Modell zu einem anderen Ergebnis kommt, beweist nicht viel, solange dieses selbst nicht verifiziert ist. Aber ich lasse den Einwand gelten, dass man nicht wirklich weiß, was hinter dem Ereignishorizont steckt.

  44. #44 Plastefisch
    22. März 2012

    @Florian Freistetter
    Aber die Energie für die Erdbebenwellen,
    welche mit dem passieren des SL durch die
    Erde ausgelöst werden, muß doch irgendwo
    herkommen. Ich nahm an das die aus der
    Bewegungsenergie des SL stammt. Aber wenn
    keine Wechselwirkung stattfindet…?

  45. #45 Florian Freistetter
    22. März 2012

    @Plastefisch: “Ich nahm an das die aus der Bewegungsenergie des SL stammt. Aber wenn keine Wechselwirkung stattfindet…?”

    Das tut sie ja auch. Aber du wolltest wissen, ob das SL Material von der Erde frisst. Das tut es nicht. Aber d.h. ja nicht, das keine Wechselwirkung stattfindet.

  46. #46 Plastefisch
    22. März 2012

    @Florian Freistetter
    Ne im Gegenteil. Ich fragte speziell nach der
    Wechselwirkung, weil ich eigentlich davon
    ausgehe, das daß SL eben nichts aus der Erde
    frisst, und auch irgendwelche Stösse mit Teilchen
    in der Erde für eine Energieübertragung nicht in
    Frage kommen, bleibt ja eigentlich nur die
    Gravitation übrig.
    (Ich fühle mich nicht von SLs bedroht. Ich
    finde das Thema nur interessant.)

  47. #47 Alderamin
    22. März 2012

    @Florian, Plastefisch

    Also die Gravitation alleine verursacht die Erdbeben? Ich kann nachvollziehen, dass eine relativ große Masse auf kleinem Raum die umgebende Materie anzieht und damit eine Erschütterung verursacht. Woher kommt die Energie? Sie kann nur aus der Bewegungsenergie kommen. Durch die Verdichtung der umgebenden Materie wirkt eine erhöhte Gravitation auf das SL zurück und bremst es ein wenig ab.

    Ist das ungefähr der Wechselwirkungsprozess, der die Energie für das Erdbeben liefert? Ansonsten wäre mir nicht klar, wie die Bewegungsenergie des SLs in Verformungsenergie umgewandelt wird.

  48. #48 Florian Freistetter
    22. März 2012

    @Alderamin: “Also die Gravitation alleine verursacht die Erdbeben? “

    Gehts jetzt wieder um Erdbeben? Als ich sagte, die Gravitation hat keine Auswirkungen, dachte ich, es war gefragt, wie die gravitative Anziehungskraft des SLs die Bahn der Erde beeinflusst.

    Ansonsten ist es natürlich die Bewegungsenergie, die die Erdbeben verursacht. Siehe Gleichung 1 im paper (https://arxiv.org/pdf/1203.3806v1.pdf), die “seismic wave equation”.

  49. #49 Alderamin
    22. März 2012

    @Florian

    Ich bezog mich auf Plastefischs Anmerkung

    Aber die Energie für die Erdbebenwellen,
    welche mit dem passieren des SL durch die
    Erde ausgelöst werden, muß doch irgendwo
    herkommen.

    und speziell den Mechanismus, wie die Bewegungsenergie in Verformungsenergie umgewandelt wird.

    Danke für den Link zum Paper. Dort steht zumindest, dass die Verformung durch den Gravitationsgradienten zu der Erdbebenwelle führt, allerdings wird der Verlust an Bewegungsenergie des SLs als vernachlässigbar betrachtet, obwohl er natürlich exakt so groß sein muss wie die Energie des Erdbebens, also vorhanden ist. Wie jetzt genau der Verlust an Bewegungsenergie zustande kommt, wird nicht erläutert.

    Interessant ist auch eine Bemerkung, dass die Kräfte in unmittelbarer Nähe des SL so groß sind, dass Steine brechen und sich erhitzen würden. Wenn also jemand unglücklicherweise von so einem Ding durchdrungen würde, wie weiter oben mal angesprochen wurde, dann würde das möglicheerweise doch etwas mehr als nur Unwohlsein verursachen. Die Chance dafür ist natürlich astronomisch klein.

  50. #50 Jack
    22. März 2012

    kann man ein SL zerstören? oder würden selbst 2 SL die mit c kollidieren zu einem größeren verschmelzen?

  51. #51 Alderamin
    22. März 2012

    @Jack

    Sehr gute Frage, hab’ mal gegoogelt. Folgendes Paper gefunden:

    https://prd.aps.org/abstract/PRD/v46/i2/p694_1

    Aus dem Abstract geht indirekt hervor, dass sie verschmelzen würden. Das Paper selbst ist leider kostenpflichtig.

    Hätte ja auch sein können, dass sie sich einfach durchdringen und wieder trennen könnten, der Ereignishorizont ist ja eigentlich nur Raum-Zeit-Krümmung. Ist aber offenbar nicht so.

    Der Autor hat auch ein Buch zu dem Thema geschrieben, allerdings ziemlich teuer.

    https://www.amazon.de/Black-Holes-Gravitational-Interactions-Mathematical/dp/0198514794

  52. #52 rnlf
    22. März 2012

    Ich finde es interessant, wie die ganze Weltuntergangshysterie sogar das Verhalten von Nichthysterikern beeinflusst. Jeder zweite scheint sich gezwungen zu fühlen, darauf hinzuweisen, dass er sich nicht bedroht fühlt sondern nur das Thema interessant findet.

    Schade eigentlich, es könnte ohne diese – nennen wir sie Schere im Kopf – viel entspannter zugehen hier 😉

  53. #53 Niels
    22. März 2012

    @Alderamin
    Schwarze Löcher können sich nicht einfach durchdringen. Beide Löcher haben schließlich Masse und verursachen Raumkrümmung, sie müssen sich also anziehen.
    Deswegen können sie nicht einfach weiterfliegen, wenn sie sich zu nahe gekommen sind.
    Vielmehr umkreisen sie sich auf immer enger werdenden Bahnen und verschmelzen schließlich in einer finalen Kollision zu einem einzigen Loch.
    Dieser Vorgang ist übrigens die stärkste Quelle für Gravitationswellen im heutigen Universum.

    Das schwarze Löcher miteinander verschmelzen ist auch schon lange anerkannter Standard. Das ist allerdings extrem schwer zu berechnen oder zu simulieren, da braucht man schon einen Supercomputer.
    https://arxiv.org/abs/gr-qc/0012079v2

    Bei den schwersten bekannten supermassiven Löchern ist meines Wissens Forschungskonsens, dass sie nur durch Verschmelzung entstanden sein können.

    Wenn zwei Galaxien verschmelzen, verschmelzen immer auch ihre zentralen schwarzen Löcher.
    Beim Galaxienhaufen Abell 400 kann man gerade den Beginn einer solchen Entwicklung beobachten.
    https://en.wikipedia.org/wiki/Abell_400

  54. #54 Niels
    22. März 2012

    Es gibt auch noch diese alpha-Centauri-Folge darüber:

    Verschmelzen Schwarze Löcher?
    https://www.br.de/fernsehen/br-alpha/sendungen/alpha-centauri/alpha-centauri-schwarze-loecher-2001_x100.html

    Hab ich allerdings noch nicht gesehen.

  55. #55 Jack
    22. März 2012

    Gut also Kollison kann kein SL zerstören, da bliebe noch Rotation – wobei ich mich grad frage ob ein Körper so schnell rotieren kann das seine Oberfläche c erreicht. auseinanderfliegen würde es davon vermutlich nicht, aber eventuell würde es sich aufblasen und seine Dichte abnehmen.
    Die letzte Möglichkeit wäre noch ein GRB der an den Polen auf das SL einwirkt, also quasi seine Entsehung umdreht und es mit soviel Energie füttert das es seine Dichte abnimmt.
    Ich nehme an die Wärmeleitfähigkeit eines SL ist gigantisch da es keine Molekularbewegung mehr gibt sondern alles quasi wie ein Atomkern (QGP) ist.

  56. #56 Alderamin
    22. März 2012

    @Niels

    Ich weiß, das SLs sich normalerweise umkreisen und verschmelzen, hier ging es ja darum, was bei einem zentralen Zusammenstoß mit fast Lichtgeschwindigkeit passiert, da hatte ich bisher noch nicht drüber nachgedacht. Der Abstract des Papers sagt ja eindeutig, dass die SLs verschmelzen.

    Wenn die beiden Singularitäten sich näher kommen als der Schwarzschildradius des größeren Objekts, dann reicht jedenfalls die Lichtgeschwindigkeit nicht mehr als Fluchtgeschwindigkeit aus, dann werden die SLs irgendwie verschmelzen. Wie sieht’s aus, wenn sich die Schwarzschildradien bei einem engen Vorbeiflug nur außen tangieren? Können sie sich dann vielleicht nochmal lösen, oder ist dann bereits die Verschmelzung unvermeidlich?

  57. #57 Alderamin
    22. März 2012

    @Jack

    Ich hab’ neulich in einer Sky & Telescope einen Artikel über die Messung der Rotation von Schwarzen Löchern gelesen, und die Aussage der Autorin war, dass alle untersuchten SLs mit der theoretischen Maximalgeschwindigkeit c rotierten (ich müsste nachschauen, wie das gemessen wurde).

    Auch Millisekunden-Pulsare rotieren schon mit einem großen Bruchteil von c (2/3 oder so). Sie sind dabei kaum größer als Schwarze Löcher. Das zerreisst sie nicht.

    Bei Schwarzen Löchern ist das ohnehin klar, dass sie die Rotation nicht zerstören kann, denn der Schwarzschild-Radius ist ja gerade der Abstand, wo auch eine Umlaufbahn mit der Geschwindigkeit c nicht mehr zum Entkommen ausreicht.

  58. #58 Jack
    23. März 2012

    @Alderamin
    Das ist eigentlich sehr einleuchtend. Wenn der Stern sich vorher dreht und dann seine Masse zum Mittelpunkt verschiebt muss seine Rotation aufgrund der Erhaltung des Drehimpulses ja steigen. Eventuell ist das auch ein Teil der Erklärung für den Plasma Jet denn vermutlich steigt die Geschwindigkeit über c wodurch die Masse in Energie konvertiert. Übrig bleibt das SL was wohl mit Materie kaum noch was gemein hat.
    Und wenn das ein unumkehrbarer Prozess ist dann wird irgendwann jegliche Materie als Singularität vorliegen. Vielleicht hilft dann nur noch der Big Rip… und aus der Singularität entsteht ein neues Universum

  59. #59 Alderamin
    23. März 2012

    @Jack

    Die Geschwindigkeit kann niemals über c steigen. Wenn Du ein großes Rad so schnell drehen würdest, dass sein Außenrand in die Nähe von c käme, dann sorgte die relativistische Längenverkürzung dafür, dass der Umfang kleiner wird, er ist dann weniger als 2*Pi*Radius, der Raum ist nicht mehr euklidisch, sondern gekrümmt. Und schwarze Löcher krümmen den Raum ja mächtig um sich herum.

    Die Jets rühren vielmehr von den Magnetfeldern einer Akkretionsscheibe her. Jets entstehen auch in den Akkretionsscheiben um junge Sterne, sog. T-Tauri-Sterne, wo gar kein Schwarzes Loch beteiligt ist und die Rotation langsamer verläuft. Sie hängen wohl damit zusammen, das aufgeheiztes Gas in der Scheibe zum Plasma wird (Elektronen und Kerne trennen sich), welches elektrisch leitend ist. Bewegte Ladungsträger erzeugen Magnetfelder. Diese werden entlang der Achse der Scheibe durch deren Rotation verdrillt und bilden so eine Art Beschleuniger, in dem Gasteilchen nach außen geschossen werden. Wenn die dann in langsameres Gas hineinrasen, entsteht Bremsstrahlung, die sich als Röntgenstrahlung bemerkbar macht.

    Schwarze Löcher werden übrigens auch nicht ewig leben, sondern sie senden Hawking-Strahlung aus: wenn ein virtuelles Teilchenpaar am Ereignishorizont entsteht (und das passiert im Vakuum laufend), kann zufällig eines der Teilchen hineinfallen und das andere nicht, bevor sie sich wieder vernichten können. Da somit Netto ein Teilchen entstanden ist, hat das Schwarze Loch durch das aufgenommene Teilchen Masse verloren und nicht etwa gewonnen. Der Prozess ist für große SLs hinreichend langsam, dass es so um die 10^80 Jahre dauert, bis sie zerfallen (im Moment reicht sogar die kosmische Hintergrundstrahlung aus, sie mehr wachsen zu lassen, als sie durch Hawking-Strahlung an Masse verlieren). Nur sehr kleine SLs zerfallen schnell und im Extremfall sogar explosiv. Sagt jedenfalls Steven Hawking, beobachtet wurde das noch nicht.

  60. #60 Wurgl
    23. März 2012

    @Alderamin: ist schon ein wenig angestaubt, aber da steht was eventuell anderes zur Rotation.
    https://www.wissenschaft.de/wissenschaft/news/272245.html

    Überhaupt etwas widersprüchlich zu anderen Aussagen, bei Wikipedia wird zum Beispiel von einer Vergrößerung des Ereignishorizonts geschrieben.

  61. #61 Niels
    23. März 2012

    @Alderamin
    Oh, sorry. Da hab ich dich komplett falsch verstanden.
    Ich hab mich schon gewundert, sonst weißt du bei diesen Sachen ja sehr gut Bescheid.

    Wenn die beiden Singularitäten sich näher kommen als der Schwarzschildradius des größeren Objekts, dann reicht jedenfalls die Lichtgeschwindigkeit nicht mehr als Fluchtgeschwindigkeit aus, dann werden die SLs irgendwie verschmelzen. Wie sieht’s aus, wenn sich die Schwarzschildradien bei einem engen Vorbeiflug nur außen tangieren? Können sie sich dann vielleicht nochmal lösen, oder ist dann bereits die Verschmelzung unvermeidlich?

    Man darf sich die Situation bei zwei derart nahe beieinander liegenden schwarzen Löchern nicht einfach linear zusammengesetzt aus den Einzelraumzeiten der beiden schwarzen Löcher vorstellen.
    Die DGLs der ART sind ja nichtlinear, die neue Raumzeit mit den zwei unmittelbar benachbarten schwarzen Löchern sieht mit Sicherheit völlig anders und sehr viel komplizierter aus als eine einfache Addition der beiden Einzelraumzeiten.
    Bei rotierenden schwarzen Löchern kommen auch noch das frame-dragging und die Ergosphäre(n) dazu, bevor man überhaupt den Ereignishorizont erreicht.

    Schau dir mal Fig.6 auf Seite 3 des oben von mir verlinkten Papers an. Das ist eine Simulation der Verschmelzung.
    Hier sind noch jeweils andere Bildausschnitte aus Simulationen von Verschmelzungen von SLs abgebildet:

    https://idw-online.de/de/news13561
    https://physics.aps.org/story/v25/st22

    Lange Rede kurzer Sinn:
    Eine äußere Berührung/Tangierung der beiden Ereignishorizonte gibt es gar nicht, weil eine Raumzeit mit zwei schwarzen Löchern anders aussieht als nur die Zusammensetzung der Einzelfälle.
    Mit der Größe der Einzel-Ereignishorizonte für allein stehende schwarze Löcher hat der Verschmelzungsprozess eher wenig zu tun.
    Sie verschmelzen schon lange, bevor sich diese Einzel-Ereignishorizonte für den alleinstehenden Fall berühren würden.

    Ich vermute aber, dass die Verschmelzung schon lange besiegelt ist, bevor sich der gemeinsame Ereignishorizont bildet.
    Für SL-Verschmelzung bin ich aber wirklich kein Experte.

    Ich weiß, das SLs sich normalerweise umkreisen und verschmelzen, hier ging es ja darum, was bei einem zentralen Zusammenstoß mit fast Lichtgeschwindigkeit passiert, da hatte ich bisher noch nicht drüber nachgedacht.

    Ich auch nicht.
    Wenn du willst, kann ich mal nachschauen, ob ich über das Institutsnetzwerk Zugang bekomme.

    Dann könnte ich dir das Paper mailen.
    Zu Lehrzwecken ist das erlaubt und du bist ja Student an meiner Uni, gell? 😉

  62. #62 Alderamin
    23. März 2012

    @Niels

    Danke für die Links und Erläuterungen.

    Gerne bin ich bei Dir Student 🙂

    Ich hatte ja schon ein Bezahl-Paper
    https://prd.aps.org/abstract/PRD/v46/i2/p694_1
    gefunden, vielleicht hast Du da zufällig kostenfreien (oder Flatrate-) Zugriff drauf?

    Meine Mail-Adresse kann Florian Dir geben, wenn er so nett wäre.

  63. #63 Alderamin
    23. März 2012

    @Wurgl

    Ich schau’ gleich zu Hause mal in dem Artikel nach. Aber, hey, 98% der Lichtgeschwindigkeit ist ja schon fast c.

  64. #64 Niels
    23. März 2012

    @Alderamin
    Klar, das Paper hatte ich ja auch gemeint.

    Ich habs jetzt der Einfachheit halber mal hochgeladen. Sag bitte Bescheid, wenn du es auf dem Computer hast, dann nehm ich es wieder runter.
    (Wenns mit diesem Anbieter nicht klappt kannst du mir gern einen anderen vorschlagen. Mit dem Hochladen hab ich wenig Erfahrungen.
    Ich hab einfach mal den ersten google-Treffer benutzt, bei dem sich nicht anmelden muss und bei dem man Dateien trotzdem wieder löschen kann.)
    https://www.filehosting.at/file/details/323837/blackhole.pdf

    Nach kurzem Überfliegen vermute ich aber, dass du damit leider nicht besonders viel anfangen können wirst.

  65. #65 Alderamin
    23. März 2012

    @Niels

    Ok, hab’ ihn. Danke. Muss jetzt erst mal abendessen.

  66. #66 Alderamin
    23. März 2012

    @Wurgl

    Ich hab’ den Artikel gefunden, Maiausgabe 2011, und nehme meine Behauptung zurück und behaupte jetzt das Gegenteil. Schwarze Löcher drehen sich mit allen möglichen Geschwindigkeiten. Die gemessenen Geschwindigkeiten lagen zwischen 0,05 und 0,99 c.

    Man misst die Rotation übrigens über den inneren Durchmesser der Akkretionsscheibe. Der ist umso kleiner, je schneller das Schwarze Loch rotiert: bei einem nicht rotierenden SL ist der innere Rand 3 Ereignishorizont-Durchmesser vom Ereignishorizont entfernt, bei retrograder Rotation mit c sind es 9 Durchmesser.

    Diesen Durchmesser kann man auch nur indirekt messen, indem man im Röntgenbereich die Eisen-Emissionslinie Fe Kɑ beobachtet, deren Breite und Verformung auf den inneren Scheibendurchmesser schließen lassen (innen geht’s schneller im Kreis).

    Ich habe aber noch das hier gefunden, wo steht, dass Supermassive Schwarze Löcher im frühen Universum mit der theoretischen Maximalgeschwindigkeit rotierten, was ältere SMSLs in unserer Umgebung nicht mehr tun. Vielleicht hatte ich den ersten Teil davon im Kopf behalten.

  67. #67 Wurgl
    23. März 2012

    Alderamin, ich hab das auch falsch gelesen. Auch das bei Wikipedia.

    Egal. In der Gegend dieser Dinger treten so seltsame Effekte auf, dass man das ohnehin kaum begreifen kann. Auch dieser Schwarzschildradius ist zwar ein Begriff nur sehe ich die Größe in Metern (oder Kilometern) einfach als Hausnummer, denn der Raum selbst ist verzerrt, somit haben unsere Längenmaße nur eingeschränkte Bedeutung.

  68. #68 Aventin
    26. März 2012

    Hat die Gravitation eine Geschwindikeit?
    Wenn ja? Wie kann die dann den Ereignisshorizont
    überwinden? Wo doch angeblich die Zeit stillstehe?
    Wenn die Gravitation sofort wirkt, überträgt sie keine Information?
    Das ja eine Information eben über das Schwarze Loch gibt?
    Wie also kann eine Information den Ereignisshorizont überwinden?

  69. #69 Aventin
    26. März 2012

    Ich denke man sollte von einem schwarzen Rand, oder Horizont reden?
    Wenn die Zeit stillsteht, endet alles, auch alle Bewegung?

  70. #70 Aventin
    26. März 2012

    Scharze löcher sollten um Planklängen grösser sein als sie sind? Oder so erscheinen?

  71. #71 Aventin
    26. März 2012

    Planklängen? Weche Auswirkungen hätte das auf die sogenannte “Howkisstrahlung”?
    Würde Planklängen den Ansatz von Steven zerstören?

  72. #72 Florian Freistetter
    26. März 2012

    @Aventin; “Hat die Gravitation eine Geschwindikeit?”

    Lichtgeschwindigkeit

    “Wie also kann eine Information den Ereignisshorizont überwinden? “

    Die Gravitation ist ein Effekt der Raumkrümmung. Da muss nichts irgendwas überwinden.

    “Planklängen? Weche Auswirkungen hätte das auf die sogenannte “Howkisstrahlung”? Würde Planklängen den Ansatz von Steven zerstören? “

    Ich habe keine Ahnung was du meinst, tut mir leid. Vielleicht formulierst du das alles noch klarer, ausführlicher und verständlicher?

  73. #73 Alderamin
    26. März 2012

    @Aventin

    Hat die Gravitation eine Geschwindikeit?
    Wenn ja? Wie kann die dann den Ereignisshorizont
    überwinden? Wo doch angeblich die Zeit stillstehe?

    Die Gravitation ist zunächst mal die Krümmung der Raumzeit, die ist statisch, die breitet sich nicht aus. Wie ein von einem Gewicht eingedelltes Gummituch. Eine Geschwindigkeit haben nur Änderungen der Gravitation. Z.B. wenn sich zwei Sterne oder Schwarze Löcher eng umkreisen, dann rühren sie sozusagen die Raumzeit um. Oder wenn ein Stern zum Schwarzen Loch kollabiert. Das erzeugt eine Welle in der Raumzeit, und die breitet sich mit Lichtgeschwindigkeit aus.

    Es muss also nichts den Ereignishorizont überwinden, um draußen Schwerkraft zu verursachen. Das Schwerefeld war ja auch schon beim Vorgängerstern da, es hat sich lediglich in einem Bereich, der vorher innerhalb des Sterns lag, vertieft, bis zum Ereignishorizont, also außerhalb desselben.

    Wenn die Gravitation sofort wirkt, überträgt sie keine Information?
    Das ja eine Information eben über das Schwarze Loch gibt?
    Wie also kann eine Information den Ereignisshorizont überwinden?

    Ein statisches Schwerefeld trägt keine Information (im Sinne der Informationstechnik, Übertragung von Bits), genau so wenig wie ein statisches elektrisches Feld, dazu müsste man es modulieren, d.h. zeitlich verändern, und das führte zur Aussendung von Wellen (Radiowellen beim elektrischen Feld). Gravitationswellen könnten Information übertragen, aber die können ein Schwarzes Loch nicht überwinden.

    Wenn die Zeit stillsteht, endet alles, auch alle Bewegung?

    Wenn etwas ins Schwarze Loch fiele und wir würden das beobachten, würde der Vorgang immer langsamer erscheinen, je näher das Objekt dem Ereignishorizont käme und dabei die Lichtwellenlänge immer länger werden. Das Objekt würde quasi am Ereignishorizont verblassen. Wir würden nicht sehen, wie es ihn durchdringt.

    Scharze löcher sollten um Planklängen grösser sein als sie sind? Oder so erscheinen?

    Verstehe ich nicht, die Plancklänge ist eine theoretische Quantelung des Raums, unterhalb derer keine räumlich Ausdehnung mehr definiert ist, und die ist noch kleiner im Verhältnis zu einem Proton, als dieses im Verhältnis zu einem Menschen. Die Singularität im Schwarzen Loch, so es sie denn gibt, könnte die Ausdehnung einer Planck-Länge haben, wenn der Raum so gequantelt wäre, was noch niemand nachgewiesen hat.

    Planklängen? Weche Auswirkungen hätte das auf die sogenannte “Howkisstrahlung”?
    Würde Planklängen den Ansatz von Steven zerstören?

    Der Mann heißt Steven Hawking. Ein gequantelter Raum würde an der Hawking-Strahlung nichts ändern, die virtuellen Teilchen, die im Vakuum spontan entstehen, legen auf jeden Fall mehr Strecke zurück, bis sie sich wieder vereinigen, d.h. auch im gequantelten Raum könnte eines in den Ereignishorizont fallen und das andere nicht. Das ist ja der Mechanismus der Hawking-Strahlung.

  74. #74 Aventin
    26. März 2012

    Gravitation keine Information?
    1 kilo 2kilo oder eine milliarde Kilo sind keine Information?

    Wird diese Info nun mit Lichtgeschwindigkeit übertragen?
    Wenn nicht, welche Folgen hätte eine unendliche Geschwindigkeit?

    Nochmal, wie kann anschaulich klar werden, warum diese Information die Schranke
    der Zeitlosigkeit überwinden kann?

  75. #75 Florian Freistetter
    26. März 2012

    @Aventin: “Nochmal, wie kann anschaulich klar werden, warum diese Information die Schranke der Zeitlosigkeit überwinden kann? “

    Ich glaube, du hast ein paar wesentliche Sachen nicht verstanden (bzw. nicht verstehen wollen). Was genau ist denn bei Alderamins ausführlichen Kommentar unklar geblieben?

  76. #76 Aventin
    26. März 2012

    @Alderamin
    Danke erstmal für die Antwort
    “Die Gravitation ist zunächst mal die Krümmung der Raumzeit, die ist statisch, die breitet sich nicht aus. Wie ein von einem Gewicht eingedelltes Gummituch. Eine Geschwindigkeit haben nur Änderungen der Gravitation. ”

    Nun, wenn der Raum sich ausbreitet, sollten das auch die statischen Zustände des Raumes?

    Die Sonne verstahlt Masse zu Energie? Diese Energie kann max mit c entweichen?
    Wie schnell ist dann die Änderung der Raumzeit?
    Und warum sollte die langsamer sein, als der Zustand vor der Steinzeit?
    Wer legt das fest? Und wie wird das festgelegt? Der Zustand der Raumzeit?

  77. #77 Aventin
    26. März 2012

    @Florian Freistetter

    Nicht verstehen wollen? Warum das? Ich habe nur Fragen, ich gerne beantwortet hätte.
    Denn die ist doch wichtig für mich, sonst nix!

    Warum machen sich die “Lochforscher” dann um ein “Informationsparadoxon” Gedanken?
    Die Menge der Entropie trägt doch auch keine Information, in dem beschriebenen Sinn?

  78. #78 Compuholic
    26. März 2012

    Verstehe Deine Frage nicht wirklich, kannst Du Die auch deutlicher machen:

    – Die Sonne verstahlt Masse zu Energie?
    Ja, und das sogar nicht zu knapp. Mehrere Milliarden kg pro Sekunde.

    – Diese Energie kann max mit c entweichen?
    Viel zu unscharf formuliert. Energie in Form vom elektromagnetischen Wellen, ja. Die breiten sich mit c aus. Energie in Form von Sonnenwind, nein. Der ist deutlich langsamer.

    – Wie schnell ist dann die Änderung der Raumzeit?
    Äh was?

    – Und warum sollte die langsamer sein, als der Zustand vor der Steinzeit?
    Siehe oben

    – Wer legt das fest? Und wie wird das festgelegt? Der Zustand der Raumzeit?
    Siehe oben

  79. #79 Aventin
    26. März 2012

    Gummituch als Beispiel ist schlecht!
    Denn hier hat die Ausbreitung einer Wirkung auch Geschwindigkeit?
    Warum ist Gummi den schneller als c?
    Warum gelingt es auch der elektischen Ladung dem Horizont zu entkammen?
    Dinge, die offentsichtlich Zeit brauchen, entkommen der Zeitlosigkeit?

  80. #80 Aventin
    26. März 2012

    @Compuholic
    Du willst meine Fragen nicht verstehen?

    Die Energie, die die Sonne unter sich lässt habe ich mit max. c beschrieben!
    Max ist eben nicht alle Energie! Warum dann der Unsinn vom Sonnenwid? Was soll das?

    Meine Frage ist ganz einfach! Wie schnell überträgt sich Gravitation?
    Wie schnell übbertagen sich deren Wirkungen?
    Und warum, wenn diese den Zeit erfordern, können diese aus Schwarzen löchern raus?

  81. #81 Compuholic
    26. März 2012

    Da ich Dich Frage immer noch nicht verstehe, antworte ich mal zur oberen Frage: “Die Menge der Entropie trägt doch auch keine Information”

    Da ist was ganz grundsätzliches durcheinandergekommen. Erst einmal macht der Begriff “Menge der Entropie” keinen Sinn. Wenn Du Entropie im Sinne von Information verwendest, ist Entropie der durchschnittliche Informationsgehalt einer Nachricht (such mal nach Shannon-Information).

    Also macht es auch keinen Sinn davon zu sprechen, dass “Entropie Information trägt”, denn Entropie ist ein Maß für Information.

  82. #82 Kallewirsch
    26. März 2012

    Gummituch als Beispiel ist schlecht!

    Ganz im Gegenteil. Diese Analogie im “2-dimensionalen” ist sogar ziemlich gut. Die Kugel in der Mitte muss nichts tun. Sie ist einfach nur da und dellt das Gummituch ein. Man darf nur nicht den Fehler machen, zuviel in diese Analogie hineinzulegen. Diese Analogie funktioniert nur dann, wenn ich genau von oben auf das Gummituch drauf schaue: Eine 2.te leichtere Kugel wird von der ersten scheinbar ‘angezogen’, so als ob die schwere Kugel eine Kraft ausüben würde. Tatsächlich existiert diese Anziehungskraft aber nicht. Es ist einzig die Form des Gummituchs, das dieses ‘aufeinander zurollen’ bewirkt.

    Und nein, die Delle entsteht nicht plötzlich. Denn auch ein schwarzes Loch entsteht ja nicht aus dem nichts, sondern es gab da Vorläufermassen, die für sich alleine gesehen noch kein schwarzes Loch darstellen. Der Raum war also auch vor der Existent des SL schon eingedellt.

    Denn hier hat die Ausbreitung einer Wirkung auch Geschwindigkeit?

    Natürlich. Wenn ich die Kugel auf dem Gummituch auf und abschwingen lasse, dann braucht es Zeit, bis sich diese Änderungen in Form von Wellen auf dem Tuch ausbreiten.

    Warum ist Gummi den schneller als c?

    Ist er doch gar nicht! Warum sollte der das sein?

    Warum gelingt es auch der elektischen Ladung dem Horizont zu entkammen?

    Auch das tut es nicht. Was du meinst ist die Hawking-Strahlung. Die entsteht genau am Horizont. Das eine Teilchen fliegt geradewegs vom Horizont weg, das andere in ihn hinein. Das, welches hineinfliegt, hat keine Chance mehr jemals zu entkommen. Das andere hingegen kommt weg.

    Dinge, die offentsichtlich Zeit brauchen, entkommen der Zeitlosigkeit?

    Du hast da ein Missverständnis. Von innerhalb des Horizonts kommt keine Information mehr heraus, ausser der dass dort Masse ist, weil diese den Raum gekrümmt hat. Das ist aber ziemlich trivial. Denn selbst dann wenn ich, um wieder auf das Gummituch zurückzukommen, 2 schwere Kugeln nicht direkt sehen kann, kann ich aus dem Verhalten des Gummituchs errechnen, wie schwer die Kugeln sind.
    Das ist aber keine Information, die aus dem Horizont herauskommt, denn die Kugeln waren ja auch vorher schon auf dem Tuch und haben es eingedellt. Durch das Zusammenlaufen haben sich nur die Dellen am Tuch verschoben und sind aufeinender zugelaufen. Soweit, bis sie sich soweit überlappen, dass es in einiger Entfernung keinen Unterschied mehr macht, ob das jetzt 2 einzelne Dellen knapp nebeneinander oder doch um 1 stärkere Delle handelt.

  83. #83 Kallewirsch
    26. März 2012

    Du willst meine Fragen nicht verstehen?

    Doch, das will er schon. Das Problem sind deine Fragen. Die sind so wirr formuliert, dass nicht klar erkennbar ist, wonach du eigentlich fragst.

  84. #84 Compuholic
    26. März 2012

    Meine Frage ist ganz einfach! Wie schnell überträgt sich Gravitation?

    Wurde doch von Alderamin bereits beantwortet: Änderungen in der Gravitation breiten sich mit c aus. Und Alderamin hat auch klargestellt, dass diese Änderungen aus schwarzen Löchern nicht rauskönnen. Wo ist also das Problem?

  85. #85 Aventin
    26. März 2012

    @ Kallewirsc

    Nein ich meinte nich die Stahlung von Stefen! Sondern eine der Haare der schwarzen
    Löcher!

    Meine Frage ist immernoch, wie kann Gravitation aus der Zeitlosigkeit des schwarzen Lochs
    entkommen?
    Wie genau funkt das?
    Wo es keine Zeit gibt, passiert;genau; nix!

  86. #86 Aventin
    26. März 2012

    @ Compuholi
    Das würde bedeuten, eine Massenzunahme Schwarzer Löcher ist nicht feststellbar?
    Es dürfte also keine superlöcher geben?
    Schon mal was vom “keine Haare” Theorem gehört?

  87. #87 Compuholic
    26. März 2012

    Nein ich meinte nich die Stahlung von Stefen! Sondern eine der Haare der schwarzen Löcher!

    Parse error…

    Gravitation aus der Zeitlosigkeit des schwarzen Lochs entkommen?

    Noch mal. Die beobachtete Zeitdehnung, wenn ein Objekt in ein schwarzes Loch fällt bezieht sich auf den außenstehenden Beobachter. Wenn Du in ein schwarzes Loch fallen würdest, bekommst Du davon erst einmal nichts mit. Für Dich vergeht die Zeit noch genauso schnell, wie auch sonst immer.

  88. #88 Aventin
    26. März 2012

    Ich habe immenochnicht gelesen wie die böse Gravitation aus dem Loch kommt?
    wollt ihr mir das nicht sagen? Oder wisst Ihr es nicht?

  89. #89 Aventin
    26. März 2012

    @Compuholic

    Die Zeit ändert sich für Betroffene? Deshalb wird das Licht so rot?
    Gar ganz rot? Nur die Gravitation nicht? Seldsam!

  90. #90 Kallewirsch
    26. März 2012

    Ich habe immenochnicht gelesen wie die böse Gravitation aus dem Loch kommt?

    Du hast immer noch nicht verstanden, dass Gravitation NICHT aus dem Loch entkommen MUSS! Laut Relativitätstheorie gibt es die ‘Kraft’ Gravitation nicht. Gravitation ist eine Verformung der Raumzeit und sonst nichts. Das SL verformt diese durch ihre Anwesenheit.

  91. #91 Aventin
    26. März 2012

    @ Compuholi
    Das Objekt sei die Gravitation! Warum bleit dir Zeit für diese nicht stehen?
    Warum kann sie also ihre Wirkung weiter haben?
    Das ist doch die Frage!?

  92. #92 Kallewirsch
    26. März 2012

    Oder wisst Ihr es nicht?

    Doch, wir verstehen das schon (so einigermassen). Das Problem ist, dass du irgendwo ein paar Schlagworte aufgeschnappt hast, und jetzt meinst du brauchst sie nur in einen möglichst kurzen Satz einzuflechten und dann ist schon alles klar.

    Die Zeit ändert sich für Betroffene?

    Das hat er gerade NICHT gesagt. Wenn du in ein schwarzes Loch fallen würdest und eine Uhr dabei hast, dann tickt die deinem Gefühl nach nach wie vor genauso schnell weiter, wie du das gewohnt bist. Für einen aussenstehenden Beobachter allerdings, der mit einem Teleskop auf deine Uhr schaut, sieht es so aus als ob deine Uhr immer langsamer ticken würde.

    Deshalb wird das Licht so rot?

    Und schon wieder das nächste Schlagwort. Wenn du ernsthaft daran interessiert bist, in die Materie einzusteigen, sollten wir uns mal darauf einigen nicht dauernd das Thema zu wechseln. Und du solltest dich erst mal auf Web-Seiten oder in Büchern schlau machen, wie das alles zusammenhängt. Diese Blog-Kommentare sind nämlich nicht wirklich ein gutes Werkzeug, um dir gewisse Dinge verständlich rüberzubringen. Da können ein paar Zeichnungen und Skizzen viel mehr zum Verständnis beitragen, also wie wenn hier jemand 5 Bildschirmseiten zum Thema schreibt.

  93. #93 Kallewirsch
    26. März 2012

    Das Objekt sei die Gravitation! Warum bleit dir Zeit für diese nicht stehen?
    Warum kann sie also ihre Wirkung weiter haben?
    Das ist doch die Frage!?

    Nein, das ist nicht die Frage. Die Frage ist, warum du nicht verstehst, dass Gravitation eine Scheinkraft ist, die sich aus der Verformung der Raumzeit ergibt. Und die muss nicht hinter dem Horizont herauskommen, denn diese Verformung war auch schon vorhanden, als das Schwarze Loch gerade zu SL wurde.

  94. #94 Aventin
    26. März 2012

    @Kallewirsch
    Was ist aber mit der Ereikniszeieit?
    Daraus kann keiner entkommen?
    Auch laut RT!

  95. #95 Aventin
    26. März 2012

    Warum ist die Gravitation eine “Scheinkfaft”?
    Welche Begründung gibt es dafür?
    Bitte mit Quelle!!

  96. #96 Kallewirsch
    26. März 2012

    Warum ist die Gravitation eine “Scheinkfaft”?
    Welche Begründung gibt es dafür?

    Weil die Krümmung der Raumzeit dafür verantwortlich ist!
    Das versuchen dir jetzt schon mindestens 3 Leute seit Stunden zu sagen und du hast es immer noch nicht begriffen!

    Bitte mit Quelle!!

    List, versteh und rechne die Relativitätstheorie nach. Das ist die Quelle.

  97. #97 Kallewirsch
    26. März 2012

    Was ist aber mit der Ereikniszeieit?
    Daraus kann keiner entkommen?

    Und? Wer behauptet das?

  98. #98 Aventin
    26. März 2012

    “Scheinkraft”
    Immer nochnicht beründet?

  99. #99 Florian Freistetter
    26. März 2012

    @Aventin: “Ich habe immenochnicht gelesen wie die böse Gravitation aus dem Loch kommt? wollt ihr mir das nicht sagen? Oder wisst Ihr es nicht? “

    Das wurde jetzt schon mehrmals beantworte. Die Gravitation ist nichts, was “aus dem Loch kommen” muss.

  100. #100 Kallewirsch
    26. März 2012

    “Scheinkraft”
    Immer nochnicht beründet?

    Ich kann nichts dafür, dass dir die physikalischen Voraussetzungen dazu fehlen, das zu verstehen.

  101. #101 Alderamin
    26. März 2012

    @Aventin

    Ich mach’ nochmal einen Versuch. Nehmen wir nochmal die Gummituchanalogie, die stellt in drei Dimensionen dar, was eine Masse in 4 Dimensionen mit der Raumzeit anstellt.

    Also, die Masse beult das Gummituch nach unten aus. Das Gummituch steht für unseren Raum, und zwar für alle drei Dimensionen. Wir können aus den 3 Dimensionen im Raum nicht heraus, und ein gedachter Beobachter in der Gummituch-Welt käme aus dem Gummituch nicht heraus. Das Tuch wäre für ihn der gesamte Raum, d.h. er würde nichts davon sehen, dass das Tuch in eine dritte Dimension ausgebeult ist.

    Wenn nun eine zweite Masse hinzukommt und an der ersten vorbei will, kann sie auch nicht aus dem Tuch heraus, sondern folgt der Krümmung des Tuchs. Sie würde normalerweise einer geraden Linie auf dem Tuch folgen, aber weil das Tuch gekrümmt ist, wird diese Linie gebogen, ggf. sogar zu einer geschlossenen Kurve, einer Umlaufbahn. Also: es gibt eigentlich keine Kraft, sondern die 2. Masse folgt nur ihrem vermeintlich geradlinigen Weg, der auf dem gekrümmten Tuch einen Bogen beschreibt Man muss im Gegenteil Kraft aufwenden, um diesen Weg zu ändern. Genau so, wie man Kraft aufwenden muss, um die Masse von einem geradlinigen Weg auf einem glatten Tuch abzulenken.

    Und das gleiche macht eine Masse (ob Schwarzes Loch oder nicht) mit der Raumzeit, sie verändert die Geometrie so, dass eine gerade Linie zu einem Kegelschnitt (Hyperbel, Parabel, Ellipse, Kreis) wird.

    Diese Verzerrung der Raumzeit kommt aber nicht aus dem Schwarzen Loch heraus, sondern sie ist einfach da. Sie ist entstanden, als sich Gas zu einem Stern zusammengefunden hat, der dann später erst zum Schwarzen Loch wurde. Beim Kollaps hat sich lediglich die Verzerrung in der Mitte, da wo vorher der Stern war, verändert, sie wurde dort zu einem steilen Trichter, der sozusagen senkrecht nach unten abfällt in einem kleinen Bereich, den wir Ereignishorizont nennen. Die Kugel, die das Gummituch eindellt, ist bei gleicher Masse zu einem kleinen Punkt geschrumpft, der das Tuch an dieser Stelle beliebig tief nach unten zieht.

    Für eine weiter außen umkreisende Masse hat sich aber nichts geändert, die Änderung fand nur innerhalb der Ausmaße des ursprünlichen Sterns ab. Daher muss nichts den Ereignishorizont durchdringen. Es bewegt sich nichts, es breitet sich nichts aus.

    Die Änderung innerhalb des Bereichs, wo vorher der Stern war, verursacht übrigens eine kleine Welle, die über das Gummituch nach außen läuft, sozusagen der “Plumps” wenn die Masse zum Schwarzen Loch kollabiert und im Gummituch versinkt. Eine Gravitationswelle in der Raumzeit. Und die breitet sich mit Lichtgeschwindigkeit aus. Die hat aber nichts mit der Schwerkraftwirkung selbst zu tun. Mit der Schwerkraft zu tun hat nur die Eindellung im Gummituch, die aus geraden Linien gekrümmte Bahnen macht.

    Wenn Du mir bis dahin folgen kannst, sehen wir nachher weiter.

  102. #102 Florian Freistetter
    26. März 2012

    @Aventin: “”Scheinkraft” Immer nochnicht beründet? “

    Merkst du eigentlich, wie unhöflich du dich benimmst? Du platz hier rein, haust eine Frage nach der anderen raus und ignorierst alle Antworten. Ich würde dir empfehlen, man in eine Bücherei zu gehen, dir dort eines der vielen populärwissenschaftlichen Bücher über Relativitätstheorie oder schwarze Löcher zu besorgen, es aufmerksam und komplett zu lesen. Dann wirst du schon mal jede Menge mehr verstehen. Und dann können wir auch weiter reden. Aber solange du hier weiter nur Halbsätze und irgendwelche Schlagwörter raushaust und dich nicht im geringsten bemühst, irgendwas aus den Antworten die du bekommst nachzuvollziehen, werden wir nicht weiter kommen.

  103. #103 Aventin
    26. März 2012

    “@Aventin: “”Scheinkraft” Immer nochnicht beründet? ”

    Merkst du eigentlich, wie unhöflich du dich benimmst? Du platz hier rein, haust eine Frage nach der anderen raus und ignorierst alle Antworten. Ich würde dir empfehlen, man in eine Bücherei zu gehen, dir dort eines der vielen populärwissenschaftlichen Bücher über Relativitätstheorie oder schwarze Löcher zu besorgen, es aufmerksam und komplett zu lesen. Dann wirst du schon mal jede Menge mehr verstehen. Und dann können wir auch weiter reden. Aber solange du hier weiter nur Halbsätze und irgendwelche Schlagwörter raushaust und dich nicht im geringsten bemühst, irgendwas aus den Antworten die du bekommst nachzuvollziehen, werden wir nicht weiter kommen. ”

    Ja habe ich verstanden!
    werde mich als Nichtwissenschaftler niemals mehr mit Wissenschaftlern anlegen!
    Sowas unfreundliche werde ich nicht mehr tun! So hereinzuplatzen!°

  104. #104 Aventin
    26. März 2012

    “Merkst du eigentlich, wie unhöflich du dich benimmst? ”
    Wer stellt diese Frage?
    Ich weiss, ich nerve, aber wie schnell ist die Schwerkraft?

  105. #105 Compuholic
    26. März 2012

    Ja habe ich verstanden!
    werde mich als Nichtwissenschaftler niemals mehr mit Wissenschaftlern anlegen!
    Sowas unfreundliche werde ich nicht mehr tun! So hereinzuplatzen!

    Nein, hast Du nicht!
    Es geht nämlich auch nicht ums hereinplatzen sondern um das ständige Ignorieren von Antworten, die man Dir bereits gegeben hat, wie Dein letzter Post wieder einmal eindrucksvoll beweist.

  106. #106 Kallewirsch
    26. März 2012

    Ich weiss, ich nerve, aber wie schnell ist die Schwerkraft?

    Die ‘Schwerkraft’ ist nicht schnell, sie hat keine Geschwindigkeit. Sie ist einfach da (wenn man die Sichtweise haben möchte, dass es sie gibt. Kann man machen, muss man aber nicht). Es sind Änderungen in den Masseverhältnissen, die sich in Form von Gravitationswellen mit c ausbreiten.

    Wenn urplötzlich die Sonne verschwinden würde (wohin auch immer), dann würde die Erde nach wie vor 8 Minuten lang den bisherigen Schwerpunkt des Sonnensystems umkreisen. So lange dauert es, bis sich die Veränderung der Massenverhältnisse bis zur Erde auswirken (und ja, das ist: mit Lichtgeschwindigkeit)

  107. #107 Kallewirsch
    26. März 2012

    Ich weiss, ich nerve, aber wie schnell ist die Schwerkraft?

    Die ‘Schwerkraft’ ist nicht schnell

    Um das zu präzisieren. Es macht keinen Sinn, einer Kraft die Eigenschaft *schnell* zu geben. Eine Kraft kann nicht schnell sein, sie ist kein materielles Objekt.

  108. #108 Aventin
    26. März 2012

    Nochmal die Frage!
    Würde die Sonne verschwinden? Ich eürde es erst in 8 Minuten merken?
    Und warum macht es keinen Sinn nach Geschwindigkeiten von Kräften zu fragen?

  109. #109 Aventin
    26. März 2012

    @Compuholic So als Beispiel
    @Kallewirsch “schwerkraft”
    Moment, bitte
    Die Scherkaft ist einfach “da”
    Die Änderungen der Selben haben eine Geschwindigkeit?
    Schwerkraft hat keine Geschwindigkeit., deren Änderung aber doch? Wie Das ?
    Was sagt der Wissenschaftler Florian dazu?

  110. #110 Alderamin
    26. März 2012

    @Aventin

    Und warum macht es keinen Sinn nach Geschwindigkeiten von Kräften zu fragen?

    Dir wurde eben erklärt, dass die Raumkrümmung die Schwerkraft ausmacht. Es gibt noch andere Krümmungen, z.B. die der Erdoberfläche. Die zwingt Dich, bei der Fortbewegung über die Erde in einem Bogen zu laufen/fliegen/fahren, um die Erde rum.

    Gegenfrage an Dich, wie ist die Geschwindigkeit der Erdkrümmung?

    Genau so wenig sinnvoll ist es, nach der Geschwindigkeit der Schwerkraft zu fragen.

  111. #111 Aventin
    26. März 2012

    Hätte es keine Auswirkungen auf das Weltmodell. wenn der Gravitation eine unendliche
    Geschindigkeit zugeschieben würde?
    Eine Unendliche Geschwindigkeit? “War schon immer Da”?
    Sofort hier, sofort da? Sofort überall?
    Welche Folgen hätte dies für unser Universum?

  112. #112 Niels
    26. März 2012

    Na ja, es ist aber schon eine legitime Frage, wie Kräfte bzw. die Austauschteilchen dieser Kräfte aus dem Ereignishorizont entkommen können.

    Zum einen ist stellt sich diese Frage natürlich für Gravitonen. Da kann man aber zur Not auf eine noch zu findende Quantengravitation verweisen.

    Zum anderen gibt es aber auch (zumindest für kurze Zeit) elektrisch geladene schwarze Löcher. Solche geladenen SLs ziehen andere Ladungen an, haben also ein elektrisches Feld, außerdem natürlich ein Magnetfeld. Sie strahlen wie jede elektrische Ladung bei Beschleunigung elektromagnetische Wellen ab.
    Wie entkommen diese Felder bzw. die Photonen dem Horizont?
    (Das Austauschteilchen der elektromagnetischen Kraft/der Quantenelektrodynamik ist ja bekanntlich das Photon.)

    Die einfache, aber völlig unbefriedigende Antwort ist natürlich: Bei der Lösung der Einsteingleichungen für elektrisch geladene SLs bekommt man Metriken, die äußere elektromagnetische Felder beinhalten.
    Eine richtig gute, anschaulichere Erklärung kenne ich auch nicht, auf einer Seite der NASA findet man zum Beispiel:

    How Do Gravitons Escape from a Black Hole?
    A black hole is a region of space-time which curves back in upon itself – presumably because of the collapse of a massive star to a size below the Schwarzschild radius. Photons or other particles don’t escape from “inside” a black hole, because the extreme curvature doesn’t allow any “escape trajectories”. Imagine particles moving along the inner surface of a ball, and trying to get outside the ball – they never could. (A black hole is not spherical like a ball however.)
    To better answer your question, let me first move away from gravitons, which we do not understand very well, to photons, which are better understood. It is known that black holes can have charge, and therefore a static electric field. The question is, how does this field escape the horizon? In that, I should remind you that the horizon is the surface through which one cannot send any signals. Now, signals (i.e. photons) involve time changing fields. Indeed no such signal can come out of the horizon. However, the electrostatic field of a charge does not convey any signal; it can therefore escape from the black hole. In technical parlance it is a space-like object, and those can cut across horizons. Or differently, in the language of excitations of the electromagnetic field (the photons)- there are transverse and longitudinal photons. The transverse ones are those we see, and travel at the speed of light. The longitudinal ones are those which “are there” in space, they do not convey any signal and are the static fields of the charges.
    Now at some level of approximation one could substitute gravitons for photons. the ones that correspond to the longitudinal part are those providing the static Newtonian (far away) fields of black holes (and all other objects). The ones producing gravitational radiation require “shaking” the gravitational lines of force and those come from outside the horizon.
    Drs. Louis Barbier and Demos Kazanas

    Finde ich persönlich jetzt aber noch nicht völlig befriedigend.
    Aber sicherlich besser als:
    Bei Anwendung der Formeln kommt das nun mal so raus.

    @Alderamin

    Ich habe aber noch das hier gefunden, wo steht, dass Supermassive Schwarze Löcher im frühen Universum mit der theoretischen Maximalgeschwindigkeit rotierten

    Kommt mir ziemlich spanisch vor. Wenn der Drehimpuls von SLs zu groß wird, verschwindet mathematisch der Ereignishorizont, man bekommt eine sogenannte nackte Singularität.
    Disappearing event horizons exist in the Kerr metric, which is a spinning black hole in a vacuum. Specifically, if the angular momentum is high enough the event horizons will disappear.
    (Aus Wiki.)

    Praktisch alle Forscher gehen davon aus, dass das “verboten” ist.
    https://en.wikipedia.org/wiki/Cosmic_censorship_hypothesis

    Wenn diese Hypothese durch Beobachtungsdaten widerlegt worden wäre, hätte man das mitgekriegt. Das wäre nämlich eine Sensation.

  113. #113 Aventin
    26. März 2012

    @ Alderamin

    Gegenfrage warum wird die Frage nach der Geschindigkeit der Gravitation
    nicht beantwotet?
    Das Motto, sie sei immer schon dar kann nicht ziehen?

  114. #114 Alderamin
    26. März 2012

    @Niels

    Wenn diese Hypothese durch Beobachtungsdaten widerlegt worden wäre, hätte man das mitgekriegt. Das wäre nämlich eine Sensation.

    Na ja, ich kann mich nur darauf beziehen, was in dem =>S&T-Artikel steht, und da steht wörtlich:

    A third group, led by Ruth Daly of Penn State University, studied 55 supermassive black holes that are producing powerful high-speed particle jets. Using a new method that she published last year, Daly estimated each hole’s spin from the jet’s power. She found a wide range of black-hole spins among her 55 objects. The most distant of them, up to 10 billion light-years away and seen when the universe was young, spin at the maximum speed theoretically possible. But closer black holes, some just a few tens of millions of light-years away in essentially the present-day universe, spin at only about 10% to 80% of their theoretical maximum speed.

    Stammt vom 5. Januar 2010.

  115. #115 Aventin
    26. März 2012

    @Alderamin
    Nein, die information, die soch ein Loch gibt, ist eben seine Masse! Und genau das
    ist eine Information? 10000 tausend und ein gramm!
    Genug Information?

  116. #116 Florian Freistetter
    26. März 2012

    @Aventin: “Ja habe ich verstanden! werde mich als Nichtwissenschaftler niemals mehr mit Wissenschaftlern anlegen! Sowas unfreundliche werde ich nicht mehr tun! So hereinzuplatzen!° “

    Ach komm, jetzt spiel nicht die beleidigte Leberwurst. Falls es dir nicht aufgefallen ist: Drei Leute haben hier lange und in vielen Kommentaren probiert, deine Fragen zu beantworten. Hier ist jeder gern gesehen, der Fragen stellen will. Ganz besonders Leute, die keine Wissenschaftler sind (die meisten die hier kommentieren sind keine). Aber wenn du überhaupt nicht auf die Antworten eingehst, sondern stur immer nur weiter die gleichen Fragen stellst, dann frag ich mich, was du dir hier eigentlich erhoffst…

  117. #117 Alderamin
    26. März 2012

    @Aventin

    Das Motto, sie sei immer schon dar kann nicht ziehen?

    Hat ja auch niemand behauptet. Die Erdkrümmung hat auch nicht immer existiert. Aber seit es die Erde gibt, gibt es die Erdkrümmung. Seit es den Vorläuferstern des SLs gibt, gibt es dessen Raumkrümmung. Und wenn er zum SL wird, ändert sich die Raumkrümmung in dem Bereich, der vorher innerhalb des Sterns war und sonst nirgends. Und die Raumkrümmung zwingt Massen und Licht auf eine gekrümmte Bahn, was wir klassisch als Kraft gedeutet hatten, obwohl es eigentlich keine ist.

    Wurde alles oben schon geschrieben.

  118. #118 Aventin
    26. März 2012

    @Florian Freistetter
    Gut, nur eine Frage “Wie schnell ist die Schwerkraft”?

  119. #119 Compuholic
    26. März 2012

    Sorry, aber langsam reißt mir echt der Gedultsfaden…

    Nein, die information, die soch ein Loch gibt, ist eben seine Masse! Und genau das ist eine Information? 10000 tausend und ein gramm!
    Genug Information?

    Bullshit. Es geht hier um Informationsübertragung. Und diese Information kommt nun mal nicht aus dem Inneren des schwarzen Loches zu uns sondern durch Beobachtungen außerhalb des Ereignishorizonts.

    Mit einem statischen Gravitationsfeld lässt sich keine Information übertragen. Genausowenig kannst Du mit einem Radiosender Information übertragen, wenn er immer nur einen Ton sendet. Seh Dir doch bitte erst einmal an, was man unter Information versteht (Hinweis: Google mal nach Shannon Information) bevor Du irgendwelche Dinge durcheinandermischt.

  120. #120 Aventin
    26. März 2012

    @Alderam
    Wie schnell hat sich das aegebreitet?
    mit c? oder langsamer?

  121. #121 Florian Freistetter
    26. März 2012

    @Aventin: “Gut, nur eine Frage “Wie schnell ist die Schwerkraft”? “

    Die Schwerkraft ist nichts, das eine Geschwindigkeit hat. Ansonsten hast du die Antwort und die zugehörigen Erklärungen auf diese Frage schon mehrmals erhalten. Gefällt dir die Antwort nicht, oder warum fragst du immer wieder nochmal?

  122. #122 Kallewirsch
    26. März 2012

    Von den bisherigen Antworten: was daran verstehst du nicht?

    So schwer ist das doch nicht, dass man einer Kraft an sich keine Geschwindigkeit zuordnen kann.
    Luftdruck beispielsweise hat auch keine Geschwindigkeit. Wohl aber eine Störung, eine Änderung des Luftdrucks. Die breitet sich mit einer bestimmten Geschwindigkeit aus. Den Effekt, dass bei dieser Ausbreitung Luftmassen bewegt werden, nennen wir Wind und der hat eine bestimmte Geschwindigkeit. Aber der Luftdruck an sich hat keine geschwindigkeit. Genauso wie ‘rot’ oder ‘blau’ keine Geschwindigkeiten haben. Wohl aber kann es zb in einer Flüssigkeit einen Farbumschlag geben, der sich mit einer bestimmten Geschwindigkeit ausbreitet.

    All diese Fragen nach der Geschwindigkeit von Farben, Drücken, Kräften und speziell von Gravitationskräften, sind sinnlose Fragen. Man kann diese Frage nicht stellen, weil die Voraussetzungen nicht stimmen. All diese Dingen kann man die Eigenschaft “schnell” nicht zuordnen.

    Und wenn du die Frage jetzt nochmal stellst, dann werd ich dich ignorieren.

  123. #123 Aventin
    26. März 2012

    @Compuholic
    Das lässt allerdings auf Information schliesen. den der Gehalt an Gewicht ist serwohl
    Information!Damit ist meine Meinung sehr wohl bewiesn!

  124. #124 Florian Freistetter
    26. März 2012

    @Aventin: “Damit ist meine Meinung sehr wohl bewiesn! “

    Ok, sag doch gleich, dass du hier keine Antworten auf deine Fragen willst, sondern nur hören möchtest, dass du Recht hast. Dann hätten wir uns die Mühe mit den Antworten sparen können…

  125. #125 Compuholic
    26. März 2012

    Das lässt allerdings auf Information schliesen.

    Tatsächlich? Dann klär mich doch bitte mal auf, wie ich mit einem Radiosender der ständig nur einen Ton sendet z.B. die Nachricht “Bullshit” übermitteln kann. Oder könnte es vielleicht sein, dass Du immer noch nicht verstanden hast, was unter Information verstanden wird?

    den der Gehalt an Gewicht ist serwohl
    Information!

    Im alltäglichen Sprachgebrauch, ja. Diese Information musste aber irgendwie an uns übermittelt werden (z.B. durch Gravitation) und darum UND NUR DARUM geht es hier. Diese Art der Informationsausbreitung ist auf Lichtgeschwindigkeit beschränkt und kommt auch nicht aus dem Inneren eines schwarzen Loches zu uns sondern einzig und allein durch Beobachtungen außerhalb des Ereignishorizonts.

    Damit ist meine Meinung sehr wohl bewiesn!

    Das einzige, was damit bewiesen ist, ist dass Du keine Ahnung hast, wovon Du redest.

  126. #126 Kallewirsch
    26. März 2012

    Das lässt allerdings auf Information schliesen.

    Keine Frage. Das ist eine Information.

    Das ist aber nicht der springende Punkt. Denn diese Masse war auch vorher schon vorhanden, ehe es zum schwarzen Loch kam.

    Unsere Sonne hat eine bestimmte Masse (nicht Gewicht). Diese Masse sorgt für eine Raumkrümmung (oder wenn du so willst eine Gravitationskraft zur Sonne hin). Diese Masse verschwindet aber nicht einfach, nur weil die Sonne plötzlich zu einem schwarzen Loch kollabiert. Wir können diese Gravitationskraft der Sonne messen und daraus ausrechnen wieviel Masse die Sonne hat. Würde die Sonne zu einem schwarzen Loch kollabieren, dann ist dieselbe Masse ja immer noch vorhanden, sie ist nur anders verteilt (konzentriert in einem kleinen Bereich). An der Raumkrümmung ändert das aber nichts, die Gravitation die die Sonne als schwarzes Loch auf die Erde ausübt ist dieselbe, wie die, als sie noch ‘Sonne’ war. Daher ist es uns nach wie vor möglich die Masse dieses schwarzen Loches zu errechnen. Diese Information kommt aber nicht aus dem Inneren des SL! Ich würde vereinfacht sagen: Sie ist ein Überbleibsel aus der Zeit bevor die Masse ein SL gebildet hat.

    den der Gehalt an Gewicht ist serwohl
    Information!

    Du meinst Masse. Weißt du, es macht nicht wirklich ein gutes Bild, wenn du physikalische Dinge auf hohem Niveau anzweifelst und mit Spitzfindigkeiten versuchst Punkte zu machen, und dann unterlaufen dir immer wieder Schnitzer, die einem Schüler der 6. Klasse im Physikunterricht keinesfalls nachgesehen werden.

  127. #127 Alderamin
    26. März 2012

    @Aventin

    Nein, die information, die soch ein Loch gibt, ist eben seine Masse! Und genau das
    ist eine Information? 10000 tausend und ein gramm!
    Genug Information?

    Nein. Das ist keine Information im Sinne der =>Nachrichtentechnik.

    Eine Information ist eine beliebige Nachricht, die ich aufschreiben kann und an einen Empfänger übermitteln möchte, nicht bloß irgendeine Messgröße. Dazu muss ich sie kodieren und auf einen Träger (Licht, meinetwegen auch Gravitationswellen) aufprägen. Beim Empfänger wird sie dann wieder dekodiert.

    Mag sein, dass Du das anders siehst, aber in der Nachrichtentechnik, Informationstheorie und Physik meint man die oben beschriebene Art von Information und nicht was Du darunter verstehst. Und sämtliche Einschränkungen der Informationsübermittlung betreffen auch nur genau die nachrichtentechnische Information und sonst keine.

    So, und einen schönen Tag noch.

  128. #128 Alderamin
    26. März 2012

    @Kallewirsch

    Stimmt, so betrachtet kann man die Masse schon als eine Information deuten, die zustande kam, als sich der Stern bildete, und die Information, dass da jetzt ein Stern ist, hat sich in Form sehr langwelliger Gravitationswellen (Frequenzen von 1/Millionen Jahre) bei der Entstehung des Sterns ausgebreitet, als sich das Schwerefeld bildet und seine Änderung sich in den Raum ausbreitete.

    Auch der “Plumps”, wenn der Stern kollabiert, verursacht eine Gravitationswelle, die einem Beobachter sagen würde: da ist jetzt ein Schwarzes Loch entstanden. Es sind aber immer nur die Änderungen des Zustands, die die Information ausmachen. Wenn ich die Gravitationswelle verpasse, dann sieht das Schwerefeld nach dem Kollaps außerhalb der Zone, wo der Stern vorher war, genau so aus wie vorher, ich kann nicht unterscheiden, ob er jetzt ein Schwarzes Loch ist oder nicht (es sei denn, ich ziehe zusätzliche Informationsquellen, etwa Licht, hinzu).

  129. #129 Aventin
    26. März 2012

    Das gar also keine Kolapsion des schwarze loches?
    Alles zur Erfindung?

  130. #130 Kallewirsch
    26. März 2012

    Stimmt, so betrachtet kann man die Masse schon als eine Information deuten

    Ich hab seine Frage eigentlich so verstanden, dass sie (in verständlichem Deutsch) lautet: Wenn keine Information aus dem SL heraus kann, woher wissen wir dann eigentlich über welche Masse es verfügt?

    Oder, was gleichwertig ist: Woher “weiß” eigentlich der Raum rund um das SL, wie stark er sich verformen muss, wenn aus dem SL keine Information raus kommt.

    Oder eine andere Formulierung: Ist nicht die Verformung der Raumzeit eine Information, die uns etwas über das SL verrät und die im Widerspruch zu der Aussage steht, dass nichts aus dem Inneren des SL den Horizont überschreiten kann?

    Und ehrlich gesagt finde ich diese Frage gar nicht mal so dumm.

  131. #131 Aventin
    26. März 2012

    @Florian Freistetter
    Wo ich duch keine Antwort bekomme?

  132. #132 Aventin
    26. März 2012

    Welche Gewwindigkeit hat die Gravitatin nun?

  133. #133 StefanL
    26. März 2012

    Aventin· 26.03.12 · 17:38 UhrDas gar also keine Kolapsion des schwarze loches?Alles zur Erfindung?

    Wow – von “‘Gravitonen’ im Schwerefeld eines SL” hin zu “wie lange dauert es noch den Versuch einer Wörter tippenden Turingmaschinensimulation zu identifizieren”… ironieexit

  134. #134 Aventin
    26. März 2012

    Einfrage Frage! Wie schnell ist die Gravitation?
    Bitte um einfache Antwort?

  135. #135 StefanL
    26. März 2012

    c

  136. #136 Aventin
    26. März 2012

    Ja Du hast mich falsch verstanden, denn du hast Masse mit Masse mit allem verwechselt!
    Masse ist mit Masse zu vergeichen! Mit sonst nix!

  137. #137 Florian Freistetter
    26. März 2012

    @Aventin: Du hast jetzt mindestens fünfmal gefragt, welche Geschwindigkeit die Gravitation hat. Verschiedene Leute haben dir erklärt, warum deine Frage so keinen Sinn macht und haben dir die Hintergründe erklärt. Wenn du hier weiter immer wieder die selbe Frage stellst, dann muss ich davon ausgehen, dass du kein Interesse an wirklich Antworten hast; das du gar nicht hier bist, um vernünftig zu diskutieren sondern muss dich als Spammer einordnen und dich rausschmeissen.

    Lies die VIELEN Antworten die du bekommen hast. Und denk darüber nach.

  138. #138 Compuholic
    26. März 2012

    Und ehrlich gesagt finde ich diese Frage gar nicht mal so dumm

    Die Frage finde ich auch nicht dumm. Im Gegenteil, ich finde sie sogar sehr interessant (und ich bin mich nicht einmal sicher, dass es da bereits eine vollständige Antwort drauf gibt). Mit Aventin werde ich diese Frage trotzdem nicht weiter diskutieren.

    Ok, erst einmal das, was wir (ziemlich) sicher wissen: Informationsübertragung geschieht immer durch die Zustandsänderung von einem Trägermedium (z.B. Modulation von Radiowellen oder Veränderung der geometrischen Anordnung von Masses). Die Geschwindigkeit der Informationsausbreitung ist auf Lichtgeschwindigkeit begrenzt.

    Ein statisches System überträgt gar keine Informationen (wie beim Beispiel des Radiosenders, der immer nur einen konstanten Ton sendet). Man könnte zwar meinen, dass hier Information übertragen wird, das gilt aber nur, wenn der Empfänger auch den Zustand “kein Ton” beobachten kann.

    Bei einem SL mit konstanter Masse kann also keine Information übertragen werden, weil es ja keine Änderung der Gravitation stattfindet. Das was wir als Masse messen ist zwar eine Information. Die Informationsquelle ist aber nicht das Innere des SLs sondern, der Raum außerhalb des Ereignishorizontes. Das diese Messungen Aufschluss über die Masse im Inneren des Ereignishorizontes geben können ist also erst einmal keine Verletzung der max. Geschwindigkeit für Informationsausbreitung.

    Interessant wird es aber, wenn sich nun die Masse verändert, wenn z.B. ein Objekt ins SL fällt. Die Masse des SLs müsste also anwachsen und damit die Raumkrümmung um das SL. Und hier ist es, wo ich unsicher bin. Wird hier wirklich Information von hinter dem Ereignishorizont übertragen?

    Ich glaube nein, denn ich als Beobachter außerhalb des Ereignishorizontes kann nie sehen, das das Objekt den EH erreicht. Für mich sieht es also aus, als ob sich die Masse asymptotisch dem EH nährt und daher ergibt sich auch für meinen Standpunkt nicht die Notwendigkeit einer Kommunikation aus dem Inneren des EHs.

    Aber das sind nur meine persönlichen Überlegungen, wenn ja jemand genaueres zu weiß, ich lasse mich gerne eines Besseren belehren.

  139. #139 Florian Freistetter
    26. März 2012

    Wer wirklich alles über schwarze Löcher und das Informationsparadoxon erfahren will (d.h. alles, was man ohne Mathematik und als Laie verstehen kann), dem kann ich dieses Buch hier nur sehr ans Herz legen: https://www.scienceblogs.de/astrodicticum-simplex/2010/09/wurmlocher-zeitmaschinen-und-ein-universum-dass-sich-selbst-erzeugt.php

  140. #140 Unwissend
    26. März 2012

    Wie schnell ist mein Bier alle ?

    sorry

    ist schon alle….

  141. #141 Wurgl
    26. März 2012

    @Compuholic

    Dieses für uns sichtbare asymptotische Erreichen des Ereignishorizonts verursacht immer wieder Hirnkrämpfe. Nach unserem Formelwerk kann wohl gar nichts diesen Ereignishorizont erreichen, zumindest nicht in unserer Zeitwahrnehmung. Somit können Schwarze Löcher nicht wachsen. Sie tun es aber offenbar doch, also stimmt da irgendwas nicht so ganz.

  142. #142 Alderamin
    26. März 2012

    Compuholic·
    26.03.12 · 19:08 Uhr

    Interessant wird es aber, wenn sich nun die Masse verändert, wenn z.B. ein Objekt ins SL fällt. Die Masse des SLs müsste also anwachsen und damit die Raumkrümmung um das SL. Und hier ist es, wo ich unsicher bin. Wird hier wirklich Information von hinter dem Ereignishorizont übertragen?

    Ich glaube nein, denn ich als Beobachter außerhalb des Ereignishorizontes kann nie sehen, das das Objekt den EH erreicht. Für mich sieht es also aus, als ob sich die Masse asymptotisch dem EH nährt und daher ergibt sich auch für meinen Standpunkt nicht die Notwendigkeit einer Kommunikation aus dem Inneren des EHs.

    Das ist mit Sicherheit richtig, am EH ist Schluss. Es wurde gezeigt, dass die Entropie eines SLs =>proportional zu seiner Oberfläche wächst, nicht etwa mit seinem Volumen, d.h. im Prinzip ist sämtliche Information über das SL im EH “codiert”.

    Und ich denke, dass ein Objekt, das in das SL fällt, genau so wie eines, =>das ein SL umkreist, Gravitationswellen aussendet und damit sozusagen die Veränderung des Gravitationsfeldes nach außen hin kund tut.

  143. #143 Adent
    26. März 2012

    @Aventin
    Sehr schnell und sehr schwer, reicht das jetzt?

  144. #144 Adent
    26. März 2012

    @Aventin
    Gegenfrage, wie grün ist Mittwoch?

  145. #145 Alderamin
    26. März 2012

    Wurgl·
    26.03.12 · 22:17 Uhr

    @Compuholic

    Nach unserem Formelwerk kann wohl gar nichts diesen Ereignishorizont erreichen, zumindest nicht in unserer Zeitwahrnehmung. Somit können Schwarze Löcher nicht wachsen. Sie tun es aber offenbar doch, also stimmt da irgendwas nicht so ganz.

    Die zufließende Masse vereinigt sich asymptotisch mit dem Ereignishorizont, wobei die Lichtwellenlänge gegen unendlich geht, d.h. das Objekt wird kurz vor dem Erreichen des EHs praktisch unsichtbar. Wir sehen es also nicht etwa am EH kleben oder so. Es wird einfach eins mit der Schwärze des EH, der dabei ja auch durch das neue Objekt wächst. So ungefähr stelle ich mir das bildlich vor. Dann klappt’s auch mit den Formeln. 😉

  146. #146 Niels
    27. März 2012

    @Wurgl

    Nach unserem Formelwerk kann wohl gar nichts diesen Ereignishorizont erreichen, zumindest nicht in unserer Zeitwahrnehmung. Somit können Schwarze Löcher nicht wachsen. Sie tun es aber offenbar doch, also stimmt da irgendwas nicht so ganz.

    In Schwarzschild-Koordinaten tritt am Schwarzschild-Radius, also dem Ereignishorizont, eine Singularität auf.
    Diese Singularität führt zu allerhand Problemen und auch zu falschen Vorstellungen.

    Das ist allerdings keine wirkliche “physikalische” Singularität sondern eine sogenannte Koordinatensingularität. Diese Singularität heißt so, weil sie nicht “echt” ist, sondern bei geeigneter Koordinatenwahl verschwindet. Sie hat also keine tatsächliche Bedeutung wie die echte Singularitäten im Zentrum des Schwarzen Loches. Die Koordinatensingularität zeigt nur, dass Schwarzschild-Koordinaten für die genaue Beschreibung der Situation am Ereingishorizont einfach eine ungeeignete Wahl sind.
    In unserem Fall sind geeignete Koordinaten zum Beispiel die Kruskal-Szekeres-Koordinaten. In diesem Koordinatensystem tritt am Ereignishorizont keine Singularität auf, da wird also nichts unendlich.
    In diesen Koordinaten sieht man sehr schön, dass ein Objekt ganz “normal” den Ereignishorizont überschreitet und in endlicher Eigenzeit die Singularität erreicht.
    (Ist das schwarze Loch groß genug, sind die Kräfte am Horizont übrigens so klein, dass man als einfallender Mensch im Raumanzug das Überschreiten des Horizontes locker überlebt, so dass man noch ne ganze Weile danach Zeit hat, den Anblick eines SL von innen zu genießen.)

    Natürlich ist es auch in Kruskal-Koordinaten so, dass für einen äußeren Beobachter Objekte, die hineinfallen, einfrieren. (Also sich immer weniger verändern, bis sie still stehen.)
    Das liegt an Folgendem:
    Wir sehen von einem hineingefallenen Objekt nur noch endlich viele Photonen, nämlich genau die, die es vor der Überschreitung des Horizontes abgestrahlt hat.
    Ein äußerer Beobachter empfängt im Lauf der Zeit aber immer weniger dieser restlichen Photonen, die zudem immer stärker rotverschoben sind.
    Die Photonen, die das Objekt in einer Sekunde Eigenzeit abgestrahlt hat, werden für den Beobachter also über einen allmählich immer größer werdenden Zeitbereich verteilt.
    Irgendwann ist beispielsweise eine Sekunde Eigenzeit des Objekts für einen äußeren Beobachter auf tausende Jahre dilatiert.
    Dadurch bekommt man den Eindruck, dass das Objekt beim Einfallen immer stärker verlangsamt wird und schließlich “einfriert”.
    (Und irgendwann verschwindet es ganz, weil die Strahlung so stark rotverschoben wurde, dass selbst die besten Instrumente sie nicht mehr messen können.)

  147. #147 Wurgl
    27. März 2012

    Niels, ja!

    Nur dann wäre es für uns doch so, dass alle Objekte die jemals in irgendein SL gefallen sind immer noch Licht zu uns aussenden (okay, kein Licht mehr, sondern extremste Langwellen). Aber eben immer noch fallen und fallen und noch ewig fallen. Somit das SL noch nicht erreicht haben.

    Dass aus der Sicht des Objektes der Fall ganz normal verläuft ist mir klar.

    Und das ist mein Problemchen.

  148. #148 Niels
    27. März 2012

    @Alderamin

    Und ich denke, dass ein Objekt, das in das SL fällt, genau so wie eines, =>das ein SL umkreist, Gravitationswellen aussendet

    Natürlich. Beschleunigte Massen strahlen immer Gravitationswellen ab.
    (Außer einer sphärisch symmetrische Massenverteilung, die sphärisch symmetrische Schwingungen ausführt. 😉 )
    Zu den ums SL kreisenden Objekten: Massen auf einer Kreisbahn sind ja bekanntlich auch beschleunigt.
    Gleichförmig bewegte Massen strahlen dagegen nicht. (Man braucht ein zeitlich veränderliches gravitatives Quadrupolmoment.)

    @Wurgl

    Nur dann wäre es für uns doch so, dass alle Objekte die jemals in irgendein SL gefallen sind immer noch Licht zu uns aussenden[…] Aber eben immer noch fallen und fallen und noch ewig fallen. Somit das SL noch nicht erreicht haben.

    Na ja, wir sehen das Objekt doch gar nicht. Wir sehen nur das Licht, das es abgestrahlt hat.

    Das ist selbstverständlich immer und überall so. In diesem Spezialfall ist der Unterschied aber eben ziemlich bedeutsam.
    Das Objekt sendet hier nämlich gar nicht für immer Licht aus, sondern nur, bis es den Horizont überschreitet.
    Auf dieses Licht wirken jetzt während seines Weges zu uns auf eine solchen Weise relativistische Einflüsse, dass wir das Objekt “immer noch fallen und fallen und noch ewig fallen” sehen.

  149. #149 GdW
    6. April 2012

    Mich würde mal interessieren, was passiert, wenn so ein “Primordial” auf seinem Weg durch die Erde auch durch einen Menschen durchrauscht…??? Würde man das überhaupt merken? Gäbe es dann auch eine Art “Erdbeben” im betroffenen Körper? Wie könnte sich so etwas wohl im Extremfall äußern? Unwohlsein, plötzliche Schmerzen? Oder gar eine Erklärung für diese angeblichen spontanen Selbstentzündungen? 😉

  150. #150 Unwissend
    6. April 2012

    @GdW

    ” Florian Freistetter· 21.03.12 · 18:08 Uhr

    @Ucuri: “Wenn so ein primordiales schwarzes Loch direkt durch einen Menschen ‘hindurchrast’, würde er etwas davon merken? ”

    Gute Frage. Mit 400 km/s ist es enorm schnell durch. Ob da Zeit für die Gravitation bleibt, etwas anzurichten? Recht gesund wird es vermutlich aber nicht sein…”

  151. #152 johann wohnlich
    argeninien
    30. Juni 2013

    warum sagt man ein schwarzes loch wenn allesterne rund sind und wie weit laesst sich materie komprimieren

  152. #153 Florian Freistetter
    30. Juni 2013

    @johann: Es heißt so, weil es kein licht reflektiert oder abstrahlt. Und wie weit sich Materie wirklich komprimieren lässt, weiß man noch nicht.

  153. #154 Tolga
    NRW
    29. Juli 2013

    Wenn ein schwarzes Loch sogar Licht anzieht, ist die Relativitätstheorie nicht damit widerlegt, weil die Gravitation eines schwarzen Loches schneller als das Licht ist?
    Oder ist es auf den Effekt zurück zu führen, dass Licht dem Raum folgt und ein schwarzes Loch diesen krümmt?

  154. #155 Dietmar
    29. Juli 2013

    @Tolga:

    Wenn ein schwarzes Loch sogar Licht anzieht, ist die Relativitätstheorie nicht damit widerlegt, weil die Gravitation eines schwarzen Loches schneller als das Licht ist?

    Das verstehe ich nicht. Die Gravitationswellen müssen das Licht doch nicht überholen.

    Oder ist es auf den Effekt zurück zu führen, dass Licht dem Raum folgt und ein schwarzes Loch diesen krümmt?

    Ja, das ist es wohl. Materie kann hineinfallen, Licht kann es nicht verlassen, Licht von anderen Objekten folgt der Krümmung und wird abgelenkt.

  155. #156 AlphaGino91
    ludwigshafen
    2. Oktober 2013

    Licht ist das schnellste das es im Universum gibt daher rechnen wir auch mit Lichtgeschwindigkeit (Lichtjahre u.s.w)
    es gibt nichts schnelleres wie Licht, Schwarze Löcher besitzen nur Ultra viel Masse. Sie Krümmen daher auch nur Raum und Zeit. Was ich mich frage…wenn ein Schwarzes loch ja Unendlich masse hat wieso verschlingt es sich nicht selbst? z.B wenn ich ein Loch Grabe dann schüttet es sich wegen des eigenen Gewichts selbst zu bis es fast voll ist.

  156. #157 AlphaGino91
    ludwigshafen
    2. Oktober 2013

    Nur um nochmal darauf zurück zu kommen was passiert wenn sich ein schwarzes Loch der Erde nähert: 1. die Erde würde sich keinesfalls anfangen schneller zu drehen (Mond zu Erde Prinzip) sie würde sich nur mit einer Seite zum schwarzen loch stellen was auch natürlich klar ist,es hat noch kein Astronom beobachtet das sich ein Stern oder Planet sich schneller gedreht hat bevor es von einem Schwarzen Loch verschluckt wurde… die Erde dreht sich auch und zieht mit ihrer Anziehungskraft den Mond an….dreht er sich deswegen??? und 2. wir würden ersticken weil das schwarze Loch zuerst die Ozonschicht und unsere Atmospähre wegsaugen würde… 3. da die Ozonschicht jetzt Fehlt kommt die Strahlung durch wodurch die leblosen Körper mensch und Tiere verbrennen würden ( kommt auf die Schnelligkeit des Schwarzen Loches an die es sich uns nähert ) der Rest Gefriert im Weltall bevor es Verschlungen wird. Astronauten die sich zu dem Zeitpunkt im Weltall befinden bekommen jedoch leider etwas mehr mit…sie würden zerrissen werden worauf es ja auf den Raumanzug ankommt…Löst er sich schnell ab in dem Moment würden auch sie innerhalb einer Sekunde Gefrieren bevor sie Verschlungen werden. Ich denke uns gehen schon vorher die Lichter aus bevor wir überhaupt mitbekommen das es richtig losgeht.

  157. #158 Florian Freistetter
    2. Oktober 2013

    @AlphaGino91: Wenn sich ein schwarzes Loch der Erde nähert, würde man das schon ein paar hundert Jahre vorher merken. Immerhin kann das ja nicht aus dem Nichts auftauchen sondern muss sich zu uns hinbewegen. Und da ein schwarzes Loch so schwer ist wie ein normaler Stern würde dessen Gravitationskraft schon in großer Entfernung für Abweichung der Bewegung der äußeren Planeten sorgen.

    Hier geht es aber nicht um stellare schwarze Löcher sonder um Mini-Löcher. Die sind so leicht, dass sie auf die Bewegung der Erde absolut keine Auswirkung haben. Ein schwarzes Loch “saugt” auch nichts weg – es ist KEIN Staubsauger: https://scienceblogs.de/astrodicticum-simplex/2010/01/12/schwarze-locher-sind-keine-staubsauger/

  158. #159 Alderamin
    2. Oktober 2013

    @AlphaGino91

    Ein Schwarzes Loch ist im Prinzip ein Loch in der Raumzeit um eine stark verdichtete Masse herum. Da die gekrümmte Raumszeit aus nichts Greifbarem besteht, kann sie auch nicht wie ein Loch in der Erde einstürzen.

    Die Masse, die Schwarzen Loch steckt, ist auch nicht unendlich, sondern entspricht der Menge Materie, die da hineingefallen ist (üblicherweise der Kern eines Sterns, 2,5 Sonnenmassen oder mehr). Die Schwerkraft des Schwarzen Lochs ist dabei genau so groß wie die eines entsprechend schweren Sterns, nur, man kommt viel näher an das im Vergleich zu einem Stern winzige Schwarze Loch heran, als an einen Stern. Z.B. hat die Sonne an ihrer Oberfläche etwa die 28fache Anziehungskraft wie bei uns auf der Erde – Du würdest dort das 28-fache wiegen, wie auf der Erde. Näher kommst Du der Sonne nicht, Du müsstest dann in sie hineintauchen, und nach innen nimmt ihre Schwerkraft wieder ab (da willst Du trotzdem nicht hin!).

    Bei einem Schwarzen Loch mit Sonnenmasse kämst Du aber bis auf 3 km an das Zentrum heran, ohne hineinzufallen. Das ist 230.000 mal näher. Entsprechend ist die Schwerkraft dort, die quadratisch mit der Entfernung abnimmt, etwa 54 Milliarden Mal so groß wie auf der Erde. Diese hohe Schwerkraft in unmittelbarer Nähe macht das Schwarze Loch so speziell. Nicht mal Licht entkommt ihm, wenn es sich ihm zu weit nähert. Aber man muss halt sehr nahe heran.

    Nur um nochmal darauf zurück zu kommen was passiert wenn sich ein schwarzes Loch der Erde nähert:

    Im Artikel ist hypothetischen, winzigen Schwarzen Löchern die Rede, die die Erde glatt druchfliegen könnten, ohne das viel passierte.

    Wenn hingegen ein Schwarzes Loch von Sternenmasse in das Sonnensystem käme, würde das geliche passieren, als wenn ein Stern der entsprechenden Masse das Sonnensystem kreuzte. Alle Planeten würden aus ihrer Bahn geworfen, das wäre schlimm genug für uns. Wenn es der Erde sehr nahe käme, würde die Erde in Stücke gerissen (mit allen Menschen und Mäusen) und sich als glühende heiße, strahlende Trümmerscheibe um das Schwarze Loch legen, aus der dann ein Teil des Materials in das Schwarze Loch hinein fiele und ein anderer Teil als Jet senkrecht zur Scheibe ausgestoßen würde. Das würde wohl nur ein paar Minuten dauern. Die Erde würde wie ein Tropfen in die Länge gezogen, zerplatzen und sich um das Schwarze Loch wickeln.

    Die Chance für einen solchen Treffer wäre aber extrem klein, da müsste das Schwarze Loch schon frontal auf Kollisionkurs mit der Erde sein, und die ist im All ein sehr kleines Ziel. Wahrscheinlicher wäre eine Änderung der Erdbahn (die z.B. elliptischer werden könnte; ganz schlecht für’s Klima!), und dass das Schwarze Loch dann wieder aus dem Sonnensystem herausflöge.

  159. #160 Alderamin
    2. Oktober 2013

    @AlphaGino91

    Entsprechend ist die Schwerkraft dort, die quadratisch mit der Entfernung abnimmt, etwa 54 Milliarden Mal so groß wie auf der Erde.

    Kleine Korrektur: sie wäre 54 Milliarden Mal so groß wie auf der Sonnenoberfläche und 1,5 Billionen Mal so groß wie auf der Erde. Hatte den Faktor 28 vergessen. 1 Billion = 1000 Milliarden = 1 Million Millionen.

  160. #161 Bullet
    2. Oktober 2013

    Kinkerlitzchen. 🙂

  161. #162 Alderamin
    2. Oktober 2013

    @AlphaGino91

    Ach, ich weiß, woher die Frage gerade jetzt kommt. Heute abend kommt so ein Schund auf RTL, hatte die Vorschau schon gesehen.

    Nein, nix davon stimmt.

  162. #163 PDP10
    2. Oktober 2013

    “Heute abend kommt so ein Schund auf RTL, hatte die Vorschau schon gesehen. “

    JA! Und Yvonne Catterfeld wird reingesaugt! 🙂

    Endlich mal eine Rolle, die ihren schauspielerischen Fähigkeiten angemessen ist …

  163. #164 Florian Freistetter
    2. Oktober 2013

    Kommt der Schund nicht erst morgen?

  164. #165 PDP10
    2. Oktober 2013

    “Kommt der Schund nicht erst morgen?”

    Stimmt.

  165. #166 Spritkopf
    2. Oktober 2013

    @Alderamin

    Heute abend kommt so ein Schund auf RTL, hatte die Vorschau schon gesehen.

    Dafür war die Filmkritik bei SpOn umso lesenswerter. 😀

  166. #167 AlphaGino91
    ludwigshafen
    4. Oktober 2013

    @Alderamin

    aha ok naja mein Wissen hab ich mir auch aus nem Buch gelesen und von Dokumentarfilmen geholt…manche sagen so und ander wieder so… es gibt zu viele die Spekulieren aber so wie du das erklärt hast is es wohl eher vieleicht… ich interessiere mich schon zeit meiner geburt für astronomie und ich erklär hier nur so wie ich das gesehen hab in doku und bücher naja aber ich bin kein experte (wäre ich aber gern =D)ich suche ja hier nur nach antworten und andere ansichten =)

  167. #168 lol
    df
    9. März 2014

    😀

  168. #169 martin
    finsderwalde
    20. März 2014

    kann son schwarzes loch auch auf der erde entstehen

  169. #170 Florian Freistetter
    20. März 2014

    @martin: Nein – und selbst wenn, dann wäre es nicht gefährlich.

  170. #171 DasKleineTeilchen
    12. April 2014

    insgesamt war das n sehr interessanter thread; aventin mag zwar tödlichst genervt haben, aber dafür kamen ein haufen unglaublich detailierter beschreibungen, die auch einige meiner, mir selbst so gar nicht bewussten fragen erschöpfenst beantwortet haben, thanx @all.

    nuhur eine frage meinerseits: neutrinos u. SLs. äquivalent mit photonen oder gibts da irgendwelche sondereffekte?

  171. #172 DasKleineTeilchen
    14. April 2014

    ja, versteh ich ja 😉 hätt ich mit den neutrinos auch selba rausfinden können. hab ick ja jetzt ooch. greetz FF.

  172. #173 thessl
    17. Juni 2014

    Alles hier mit gelesen und ich fand es sehr interessant…. 🙂 vieles wurde gut erklärt, sodass ich es verstand, obwohl ich mich nie wirklich mit dem thema befasst habe- und ich bin neugierig geworden auf meeeeehr 😉 ^^

  173. #174 Andreas
    Deutschland
    30. November 2014

    @ florian: habe diese seite/nachricht gefunden https://www.tempolimit-lichtgeschwindigkeit.de/graum/graum3.html.
    Was ist da dran?
    Kann uns dieses schwarze loch gefährlich werden?
    Sind schwarze löcher bekannt die für die erde gefährlich werden könnten,

  174. #175 Florian Freistetter
    30. November 2014

    @Andreas: “Sind schwarze löcher bekannt die für die erde gefährlich werden könnten,”

    Nein. Schwarze Löcher sind nicht gefährlich. Die sind so weit entfernt wie die Sterne; noch viel weiter sogar.

  175. #176 Frantischek
    30. November 2014

    Nebenbei handelt es sich bei dem Link um einen hypothetischen Bericht von einer Seite die Lehrmittel anbietet.
    Die Seite stammt aus dem Jahr 2004, der “Artikel” aus dem Jahr 2012.
    Das ist wohl sowas wie eine Aufgabe für Schüler:
    “Was wäre wenn man in der Zukunft ein schwarzes Loch in der Nähe des Sonnensystems entdecken würde.”

    Wobei Nähe relativ ist. 12 Lichtjahre sind ja ein ganz gutes Stück…

  176. #177 Andreas
    deutschland
    30. November 2014

    @florian: und um was für ein schwarzes loch handelt es sich in dem bericht?
    Steht ja immerhin dass es sich dem sonnensystem mit 240km/s nähert??

  177. #178 Andreas
    Deutschland
    30. November 2014

    @ frantischek: wegen der distanz gebe ich dir recht! 😉

  178. #179 T
    30. November 2014

    Andreas! Warum auch immer, der Verfasser dieser Seite hat sich zwei Texte in zwei nicht existierenden Zeitungen ausgedacht, um auf ironische Weise zu veranschaulichen, dass 12 Lichtjahre eine sehr große Entfernung sind. Ein solches schwarzes Loch gibt es nicht. Es ist eine Denkaufgabe und du musst dringend an deiner Medienkompetenz arbeiten. Und an deinen Rechenfähigkeiten: Wie lange braucht ein 12 Lichtjahre entfernter Körper zur Erde, wenn er mit 240 km/s unterwegs wäre?

  179. #180 JaJoHa
    30. November 2014

    @Andreas
    c=299792km/s, v=240km/s
    Mit c brauchst du 12 Jahre, Dreisatz sagt t=12Jahre*299792/240
    Das sind grob 10.000 Jahre, die genauen Zahlenwerte kannst du leicht ausrechnen.
    Bei 240km/s braucht man sich meistens keine Gedanken um Relativitätstheorie machen.

  180. #181 Andreas
    Deutschland
    30. November 2014

    @T: soso, fands schon auch komisch dass in keiner anderen seite irgendwas über dieses schwarze loch zu finden war. Komisch auch dass im text rehcts unten steht dass es sich mit dieser geschwindigkeit nähern soll, aber keine gefahr davon ausgehen kann laut dem “experten”…hab ehrlich gesagt nicht überprüft ob es die zeitungen gibt oder nicht.

    @jajoha: soweit war ich mit meiner mathematik auch gekommen, aber danke für die info und die antwort.

    @florian: deine meinung?

    Danke und beste grüsse

  181. #182 Florian Freistetter
    30. November 2014

    @Andreas: Das schwarze Loch in dem Bericht GIBT ES NICHT. Das sind Unterrichtsmaterialien die nichts mit der Realität zu tun haben. Und 240km/s sind zwar schnell. Aber das Weltall ist GROSS! Ein sich näherendes schwarzes Loch würde man schon ein paar 100 Jahre bemerken, bevor es da ist. Hör auf, dir über solche Sachen Sorgen zu machen. Schwarze Löcher sind ungefährlich für die Erde.

  182. #183 Auto-Cogitator
    24. Juli 2016

    Was ich nicht verstehe: Ein schwarzes Loch von der Masse eines Asteroiden müsste doch von der viel größeren Masse der Erde schwerkraftmäßig eingefangen werden. Wenn es zu klein ist, um mit der Materie der Erde (nenneswert) zu kollidieren, so müsste es aufgrind seiner kinetischen Energie vielleicht erst über den Erdmittelpunt hinausschießen, dann aber von der Anziehungskraft der Erde abgebremst werden, irgendwann wieder auf den Erdmittelpunkt zurückfallen, und nach ein paar Pendelbewegungen im Erdmittelpunkt zur Ruhe kommen. Oder habe ich da einen Denkfehler drin?

  183. #184 Auto-Cogitator
    24. Juli 2016

    Wahrscheinlicher als eine geradlinige Pendelbewegung wäre wohl eine Umkreisung des Erdmittelpunkts in immer engeren Bahnen.

    Aber ich sehe gerade: Die im Artikel angegebene Geschwindigkeit des primordialen schwarzen Lochs wäre deutlich über der Fluchtgeschwindigkeit von der Erde gemäß Wikipedia.

    Aber wenn man jetzt mal annimmt, es wäre eines mit kleinerer Geschwindigkeit als der Fluchtgeschwindigtkeit?

  184. #185 Alderamin
    24. Juli 2016

    @Auto-Cogitator

    Was aus dem Unendlichen kommend mit Ausgangsgeschwindigkeit 0 auf die Erde fällt, kommt dort durch die Fallbeschleunigung der Erde automatisch mit Fluchtgeschwindigkeit an. Das ist symmetrisch zu der Situation, etwas von der Erde mit Fluchtgeschwindigkeit ins Unendliche zu befördern, wo die Geschwindigkeit dann gegen 0 geht.

    Da im All nichts in Ruhe ist, wird die Auftreff-Geschwindigkeit in der Realität sogar etwas über der Fluchtgeschwindigkeit liegen.

  185. #186 Auto-Cogitator
    25. Juli 2016

    Kann ich nicht nachvollziehen. Praktisch gesehen dürfte doch die Anziehungskraft der Erde auf weit (gar unendlich weit) entfernte Objekte angesichts der übrigen Massen im Universum in allen Richtungen vernachlässigbar sein.

    Überlegung: Wenn nahe der Erde etwas den Planeten Durchdringendes mit Geschwindigkeit 0 beginnend auf den Erdmittelpunkt beschleunigt wird, so wirkt nach dem Durchgang durch diesen die Schwerkraft mit gleichem Betrag als Bremskraft, bis sich die Bewegung wieder umkehrt. Auch kleinste Reibungsverluste müssten dann irgendwann dazu führen, dass das Objekt im Erdmittelpunkt zur Ruhe kommt.

    Mal mit hoffentlich nicht zu naiv zusammengekratzter Schulphysik gerechnet für ein Objekt, dass mit Geschwindigkeit 0 beginnend in den Erdmittelpunkt fällt:

    s = 0,5 g t²

    also:

    t = Wurzel (s / 0,5 g)

    Mit g = 9,81 und s = Erdradius = 6371000 m, sowie die Tatsache vernachlässigend, dass die Schwerkraft im Inneren der Erde bis zum Mittelpunkt abnimmt:

    v = g t = 9.81 Wurzel (6371000 / 4,905) = 11180,296 m/s = 11,18 km/s

    Fluchtgeschwindigkeit von der Erde laut Wikipedia: 11,2 km/s

    (Erstaunliches nahes Ergebnis der Rechnung – wohl kein Zufall?)

    Aber wenn man nun doch berücksichtigt, dass die Beschleunigung bis zum Erdmittelpunkt (bis auf null?) abnimmt, wäre das reale Ergebnis unter der Fluchtgeschwindigkeit. Ich kann das nicht rechnen, also sagen wir mal, das reale Ergebnis wäre 8 km/s. Dann könnte ein Objekt mit der Anfangsgeschwindigkeit von 0 – 3 km/s die Erde nicht wieder verlassen.

    Soweit richtig?

    Dann hätte ich gern gewusst:

    1. Rein hypothetisch: WENN ein primordiales Schwarzes Loch unter Fluchtgeschwindigkeit (oder genauer: mit einer Geschwindigkeit relativ zur Erde, die auch nach der Beschleunigung durch die Erdschwerkraft nicht die Fluchtgeschwindigkeit erreicht) auf die Erde trifft, müsste es sich dann nicht so verhalten, wie in meinem ersten Beitrag beschrieben?

    2. Ist es denkbar (wenn auch nicht wahrscheinlich), dass ein primordiales Schwarzes Loch eine solch geringe Geschwindigkeit (relativ zur Erde) hat? (Vielleicht, weil ein Galaxienhaufen in entgegengesetzter Richtung zur Erde auf das PSL auf dessen Weg durchs Universum einen relativ zur Erde abbremsenden Einfluss hatte?)

  186. #187 Alderamin
    25. Juli 2016

    @Auto-Cogitator

    Überlegung: Wenn nahe der Erde etwas den Planeten Durchdringendes mit Geschwindigkeit 0 beginnend auf den Erdmittelpunkt beschleunigt wird, so wirkt nach dem Durchgang durch diesen die Schwerkraft mit gleichem Betrag als Bremskraft, bis sich die Bewegung wieder umkehrt.

    Ja, wenn es in Erdnähe die Geschwindigkeit 0 hätte, und dann fiele das Objekt unter Vernachlässigung von Reibung auch wieder an einen Punkt, der genau so weit weg von der Erde wie der Ausgangspunkt wäre, denn die Kraft, die das Objekt beim Entfernen bremst ist in jeder Entfernung genau so groß wie die Kraft, die es beim Herunterfallen in der entsprechenden Entfernung beschleunigt.

    Damit etwas sich der Erde nähern kann, muss es aber anfangs eine Geschwindigkeit auf die Erde zu gehabt haben, und zwar schon in großer Entfernung, sonst wäre es nie hier angekommen (die Sonne wird schon mit ihrer Schwerkraft dafür sorgen, dass diese Geschwindigkeit in relativer Erdnähe mehr als 17 km/s beträgt). Und genau mit dieser Geschwindigkeit wird es die entsprechende Entfernung wieder erreichen. Es wird nicht wieder von der Erde zurück geholt, wenn es in der Erde nicht sehr viel Impuls und Bewegungsenergie an diese abgeben kann (wie etwa ein Meteoroid, der in der Atmosphäre abgebremst wird).

    Mit g = 9,81 und s = Erdradius = 6371000 m, sowie die Tatsache vernachlässigend, dass die Schwerkraft im Inneren der Erde bis zum Mittelpunkt abnimmt:

    v = g t = 9.81 Wurzel (6371000 / 4,905) = 11180,296 m/s = 11,18 km/s

    Nee, gerade so einfach geht’s nicht, die Schwerkraft der Erde nimmt außerhalb derselben mit 1/r² ab, da müsstest Du mit Newtons Gravitationsgesetz rechnen. Aber solange die Kräfte bei An- und Abflug symmetrisch sind, kommt in beiden Fällen heraus, dass das Objekt wieder auf die gleiche Entfernung kommen wird. Also aus dem Unendlichen wieder ins Unendliche.

    1. Rein hypothetisch: WENN ein primordiales Schwarzes Loch unter Fluchtgeschwindigkeit (oder genauer: mit einer Geschwindigkeit relativ zur Erde, die auch nach der Beschleunigung durch die Erdschwerkraft nicht die Fluchtgeschwindigkeit erreicht) auf die Erde trifft, müsste es sich dann nicht so verhalten, wie in meinem ersten Beitrag beschrieben?

    Nur bei einer Ausgangsgeschwindigkeit von weniger als Fluchtgeschwindigkeit in der entsprechenden Entfernung. Aber:

    2. Ist es denkbar (wenn auch nicht wahrscheinlich), dass ein primordiales Schwarzes Loch eine solch geringe Geschwindigkeit (relativ zur Erde) hat?

    Nein. Wie gesagt, alleine die Sonnenschwerkraft wird es auf die Fluchtgeschwindigkeit der Sonne auf Höhe der Erdbahn beschleunigen, etwa 47 km/s. Wenn es der Erde frontal entgegen käme, wäre die Geschwindigkeitsdifferenz dann 77 km/s, bei einer Annäherung von hinten immer noch 17 km/s. Und dazu käme dann die Erdbeschleunigung. Die einzige Chance wäre, dass es schon immer in einer Umlaufbahn um die Sonne gewesen wäre und durch Wechselwirkungen mit der Erde, anderen Planeten und dem Mond auf eine Bahn ähnlich der Erdbahn gelangt wäre. Aber selbst dann wäre die Annäherung an die Erde nahezu so, wie eine mit ruhender Anfangsgeschwindigkeit aus dem Unendlichen (eben weil die Schwerkraft der Erde rasch abfällt, dass das annähernde Gravitationspotenzial des “Unendlichen” in der Praxis nicht sehr weit weg ist – etwa die Hill-Sphäre der Erde mit 1/100 AE = 1,5 Millionen km Radius; das ist im Sonnensystem nix). Ein “Einfangen” eines Objekts ist ganz schön kompliziert, bedarf immer eines dritten Körpers (etwa der Sonne) und wird dann eher ein Einfangen in einem weiten Orbit sein, aus dem es auch wieder verloren gehen kann, als ein direkter Zusammenstoß.

  187. #188 Auto-Cogitator
    27. Juli 2016

    Zunächst ein Nachtrag: Dass das Ergebnis meiner Rechnung so nahe an der Fluchtgeschwindigkeit lag, war wohl doch Zufall, denn bei einem anderen Planeten mit gleicher Masse aber anderer Dichte und somit anderem Radius würde ja etwas anderes herauskommen, die Fluchtgeschwindikeit wäre aber bei gleicher Masse die selbe.

    “Nee, gerade so einfach geht’s nicht, die Schwerkraft der Erde nimmt außerhalb derselben mit 1/r² ab, da müsstest Du mit Newtons Gravitationsgesetz rechnen.”

    Die Vorausssetzung bei der Rechnung war ausdrücklich “nahe der Erde”, also gerade ohne nennenswerten Abstand.

    “Aber solange die Kräfte bei An- und Abflug symmetrisch sind, kommt in beiden Fällen heraus, dass das Objekt wieder auf die gleiche Entfernung kommen wird.”

    Völlig symmetrisch sind sie nur bei Anfangsgeschwindigkeit 0. Und schon kleinste Reibungsverluste führen zu Asymmetrie und zur Annäherung and den Erdkern. Wie gezeigt sogar bei Geschwindikeiten über 0, da ab dem Eintauchen in die Erde die Schwerebeschleunigung bis zum Mittelpunkt immer schwächer wird, was bei der Rechnung nicht berücksichtigt wurde.

    “Nur bei einer Ausgangsgeschwindigkeit von weniger als Fluchtgeschwindigkeit in der entsprechenden Entfernung.”

    Na eben. Genau das war das Fall-Szenario. Damit hättest du dir die ganzen anderen Ausführungen davor zu anderen Szenarien sparen können. Also meine Frage 1. hast du mit “ja” beantwortet.

    Nun zur dann brisant werdenden Frage 2.

    Ich schrieb:

    “Praktisch gesehen dürfte doch die Anziehungskraft der Erde auf weit (gar unendlich weit) entfernte Objekte angesichts der übrigen Massen im Universum in allen Richtungen vernachlässigbar sein.”

    Und:

    “st es denkbar (wenn auch nicht wahrscheinlich), dass ein primordiales Schwarzes Loch eine solch geringe Geschwindigkeit (relativ zur Erde) hat? (Vielleicht, weil ein Galaxienhaufen in entgegengesetzter Richtung zur Erde auf das PSL auf dessen Weg durchs Universum einen relativ zur Erde abbremsenden Einfluss hatte?)”

    In deiner Schilderung gibt es nach wie vor die Erde, die auf das PSL schon in unendlicher Entfernung einen maßgeblichen Einfluss gehabt habe, das überzeugt mich nach wie vor nicht. Auch auf mögliche abbremsende Schwerkrafteinflüsse durch andere Massen bist du nicht eingegangen.

    Gewichtiger scheint mir das Argument, dass schon die Sonne für eine Beschleunigung über die Fluchtgeschwindikeit von der Erde sorge. Du kommst dabei auf eine Mindestgeschwindigkeit von 17 km/s, das wäre in der Tat 5,8 km/s über der Fluchtgeschwindigkeit. So richtig beruhigend weit davon weg wäre das aber nicht. Was ist, wenn das PSL innerhalb des Sonnensystems anderen gleich schweren und deutlich schwereren Körpern (Asteroiden, Planeten) derart ungünstig begegnet ist, das sich der Geschwindikeitsverkor in Richtunmg zur Erde um diese 5,8 km/s abgebaut hat?

    Konkretes Szenario: Das PSL kommt von außern ins Sonnensystem in Richtung auf die Erde. Unterwegs sind die Bahnbewegungen der Planeten, denen es begegnet, gerade so ungünstig, dass deren Schwerkraft im Vorbeiflug auf ds PSL in Richting auf die Erde eine abbremsende Wirkung hat. Das scheint mir doch debnkbar.

    Diesmal bitte bei dem angegebenen Szenario bleiben und nicht wieder erst lang und breit andere Teufel an die Wand malen, die sich leichter erlegen lassen. 😉

    Dass das Szenario von geringer Wahrscheinlichkeit ist, sei attestiert. Es geht um Denkbarkeit.

    (Die Entstehung intelligenten Lebens ist auch von grotesker Unwahrscheinlichkeit, dennoch ist es bei der grotesken Anzahl an Sternsystemen im Universum mindestens einmal passiert.)

  188. #189 Bullet
    27. Juli 2016

    Du kommst dabei auf eine Mindestgeschwindigkeit von 17 km/s, das wäre in der Tat 5,8 km/s über der Fluchtgeschwindigkeit. So richtig beruhigend weit davon weg wäre das aber nicht.

    Ach? Wie “weit davon weg” wäre denn “beruhigend”? Knapp 6 km/s sind in einem durch Gravitation geformten System, das ohne Antriebe technologischer Art auskommt, eine ganze Menge. Weil: fliegt weg, kommt nie wieder. Das ist wie eine Glasscheibe, die mich vor dem fiesen Raubtier im Zoo trennt: sind auch nur 5 mm Glas. Völlig ausreichend, egal wie groß die Zähne sind.

    Konkretes Szenario: Das PSL kommt von außern ins Sonnensystem in Richtung auf die Erde. Unterwegs sind die Bahnbewegungen der Planeten, denen es begegnet, gerade so ungünstig, dass deren Schwerkraft im Vorbeiflug auf ds PSL in Richting auf die Erde eine abbremsende Wirkung hat. Das scheint mir doch debnkbar.

    Ach so. Ja klar: ich baue mir ein Szenario zusammen, in dem alle siebenhundertdreiundachtzig Störgrößen gerade die korrekte dreiundzwanzigste Stelle hinterm Komma haben, die sie brauchen, um mein schönes Szenario nicht implodieren zu lassen. Das ist unlautere “Argumentation”. (Und nebenbei: entscheide dich doch mal: wenn es von außen kommend in Richtung Erde unterwegs ist, dann könnte es vielleicht abgebremst werden, dann aber wäre es nicht mehr in Richtung Erde unterwegs. Oder aber es wäre erst durch die Bahnablenkungen innerhalb des Sonnensystems auf Erdkurs gebracht – dann ist es nicht mehr abgebremst.)

  189. #190 Auto-Cogitator
    28. Juli 2016

    Welche Argumentation ist unlauter? Eine Argumentation, die ausdrücklich darauf hinweist, dass das beschriebene Szenario denkbar, aber “grotesk unwahrscheinlich” ist? Oder eine, die ein noch so unwahrscheinliches, aber denkbares Szenario, das keine physikalischen Gesetze verletzt, für unmöglich erklärt?

    Bei deiner Sichtweise wäre es wie gesagt genau so “unlauter argumentiert”, angesichts der grotesk geringen Wahrscheinlichkeit darauf hinzuweisen, dass im Rahmen einer biologischen Evolution intelligentes Leben entstehen kann. Damit lieferst du zum Beispiel Munition für Kreationisten.

    Wieso könne das PSL nach Abbremsung nicht mehr auf Erdkurs sein? Das ist doch Quatsch. Es könnte ohne Richtungsänderung abgebremst werden (unwahrscheinlich) oder mit Richtungsänderung. Durch eine Richtungsänderung aufgrund der Abbremsung könnte es, wenn es vorher auf Erdkurs war, die Erde verfehlen, und wenn es vorher nicht auf Erdkurs war, durch die richtungsändernde Abbremsung gerade erst auf Erdkurs gebracht werden.

    Es bleibt somit nach wie vor unwahrscheinlich, aber denkbar, so dass es in Wahrheit gerade unlauter argumentiert wäre, es für unmöglich zu erklären.

    Es gibt ja Spekulationen, es gäbe eine unendliche Menge ausgedehnter, mit Materie angefüllter Raumzeit – entweder, weil schon unser Universum unendlich sei, oder aufgrund unendlich vieler Paralleluniversen (Viele-Welten-Deutung der Quantentheorie) usw. Wenn es aber Unendlichkeit gibt, so gilt: Jedes Ereignis mit einer Wahrscheinlichkeit größer null ereignet sich in der Unendlichkeit unendliche Male. Also auch grotesk unwahrscheinliche Ereignisse. Dann gibt es auch unendlich viele Planeten mit intelligentem Leben und unendlich viele Planeten, die sich abgebremste PSL einfangen.

  190. #191 Spritkopf
    28. Juli 2016

    Dann gibt es auch unendlich viele Planeten mit intelligentem Leben und unendlich viele Planeten, die sich abgebremste PSL einfangen.

    Wir werden alle stääärbön!!!einself!

  191. #192 Kyllyeti
    28. Juli 2016

    Noch schlimmer:
    Es sind auch noch unendlich viele golgafrinchamische Rettungsraumschiffe unterwegs!

  192. #193 Bullet
    28. Juli 2016

    @Auto-Cogitator:

    Es könnte ohne Richtungsänderung abgebremst werden (unwahrscheinlich)

    Nicht unwahrscheinlich: unmöglich.

    Bei deiner Sichtweise wäre es wie gesagt genau so “unlauter argumentiert”, angesichts der grotesk geringen Wahrscheinlichkeit darauf hinzuweisen, dass im Rahmen einer biologischen Evolution intelligentes Leben entstehen kann.

    What? Es ist auch “grotesk unwahrscheinlich”, daß jemand einen Sechser mit Superzahl im Lotto hat. Oh. Passiert ja trotzdem dauernd.
    Außerdem ist die Entstehung “intelligenten Lebens” (was das sein soll, weiß noch keiner) nicht Ergebnis einer “grotesk geringen Wahrscheinlichkeit”. Als Analogie: klar ist es “grotesk unwahrscheinlich”, mit 1 Million Würfeln mit einem Wurf nur Sechsen zu würfeln. Aber du machst deinen ersten Wurf, sortierst alle Sechsen aus und würfelst weiter, wiederum beim nächsten Wurf die Sechsen zu den bereits gewürfelten Sechsen hinzulegend, und so weiter. Die Wahrscheinlichkeit, auf diese Weise irgendwann! eine Million Sechsen zu haben, ist nicht “grotesk wenig wahrscheinlich”, sondern etwa 1.

    Und dann noch der letzte Absatz mit den unendlichen Sachen. Der beginnt mit:

    Es gibt ja Spekulationen […]

    Das ist schön, und spekulieren darf jeder, aber das juckt niemanden. Zumal du Dinge erwähnst, die so nicht stimmen: zuerst ist unser Universum garantiert nicht unendlich. Es hatte einen Anfang, kann also nicht unendlich groß sein. Aus demselben Grund kann sich es dank der Planck-Zeit auch nicht in unendlich viele Universen aufgespalten haben. Zusätzlich gibt es den berühmten Vergangenheits- und Zukunftslichtkegel: alles, was außerhalb dessen liegt, kann niemals eine Auswirkung auf das uns prinzipiell zugängliche Universum haben. Mit anderen Worten: Solange wir in der Milchstraße bleiben, ist die Ausdehnung unseres Universums, obschon nicht konstant, jedoch bereits rechnerisch begrenzt. Unendlichkeit existiert daher nicht.

  193. #194 Auto-Cogitator
    28. Juli 2016

    Abbremsung ohne Richtungsänderung ist sehr wohl möglich: Planet 1 bremst ab, Richtung ändert sich. Planet 2 bremst ab, Richtung ändert sich genau entgegengesetzt, damit wieder Ursprungsrichtung. Unwahrscheinlich, aber nicht unmöglich.

    Aber wie gesagt kann eine oder mehrere abbremsende Richtungsänderung(en) auch erst den Erdkurs ergeben.

    Die Entstehung intelligenten Lebens ist viel unwahrscheinlicher, als du es in dem Würfelbeispiel darstellst, denn so ereignet sich Evolution nicht. In den vier Milliarden Jahren Evolution auf diesem Planeten wurde Intelligenz in menschlichem Ausmaß nur einmal erfunden, das Fliegen als andere Extremleistung dagegen vier mal völlig unabhängig voneinander (Fluginsekten, Flugsaurier, Vögel, Fledermäuse).

    Es ist völlig offen, ob das Universum unendlich ausgedehnt ist oder nur endlich. Beide Möglichkeiten sind gleich spekulativ. Das Universum hatte einen zeitlichen Anfang, kann aber auch da schon unendlich ausgedehnt gewesen sein. Dazu gibt es sogar eine Folge im Podcast zu diesem Blog: https://www.youtube.com/watch?v=Nz_ZJUxNVH8

    Mir ist jegliche Form von Unendlichkeit (sei es ein unendliches Universum, eine unendliche Zahl an Universen oder eine unendliche Zahl an ineinander verschachtelten Simulationen von Universen) auch zutiefst unsympathisch – warum, habe ich in einem Kommentar zum verlinkten Video geschrieben (Mein Youtube-Name: NuntiusLegis). Aber leider schert sich die Natur nicht darum, was irgendjemand sympathisch oder unsympathisch findet.

    Unendlichkeit ist also (leider) genau so möglich wie Endlichkeit. Und das hieße: Es gibt nicht nur unendlich viele Planeten, die sich grotesk unwahrscheinlich abgebremste PSL einfangen, sondern sogar unendlich viele exakte Kopien unserer Erde mit uns allen als Bewohnern, die dieses Schicksal ereilt. Und unendlich viele exakte Kopien von uns, denen noch viel Schlimmeres und Leidvolleres widerfährt. Im Falle von Unendlichkeit ist das absolut zwingend. Und die Chancen auf diese unendlichen Höllen für unendlich viele Doppelgänger von uns allen stehen nach derzeitigem Kenntnisstand wohl leider fifty-fifty.

    Ich plädiere deshalb immer dafür, sich wenigstens nicht ohne Not vorzeitig auf die Uenendlichkeit festzulegen, so lange die Daten auch eine endliche Welt hergeben. Ich denke, die meisten Anhänger der Unendlichkeit, etwa der Viele-Welten-Interpretation, haben die infernalischen Folgen nicht hinreichend gründlich bedacht.

  194. #195 Bullet
    29. Juli 2016

    Abbremsung ohne Richtungsänderung ist sehr wohl möglich: Planet 1 bremst ab, Richtung ändert sich. Planet 2 bremst ab, Richtung ändert sich genau entgegengesetzt, damit wieder Ursprungsrichtung. Unwahrscheinlich, aber nicht unmöglich.

    Das sagst du genau so lange, bis du herausfindest, daß eine Abbremsung im Schwerefeld eines Planeten nur funktioniert, wenn der Kurs des Objektes danach wieder nach draußen geht…

    Die Entstehung intelligenten Lebens ist viel unwahrscheinlicher, als du es in dem Würfelbeispiel darstellst, denn so ereignet sich Evolution nicht.

    Ach, nicht? Das kannst du bestimmt irgendwie unterfüttern, wa?

    In den vier Milliarden Jahren Evolution auf diesem Planeten […]

    Wäre mir neu, daß Evolution seit 4 Milliarden Jahren stattfindet. Wikipedia:

    Die ältesten eindeutigen Lebensspuren auf der Erde sind 1,9 Milliarden Jahre alte Fossilien aus der Gunflint-Formation in Ontario, die Bakterien oder Archaeen gewesen sein könnten.

    Alles andere ist Spekulation.

    […] wurde Intelligenz in menschlichem Ausmaß nur einmal erfunden

    Oh. Ein Wisser. Das ist aber schön. Du weißt ja nicht einmal, was Intelligenz ist.

    Es ist völlig offen, ob das Universum unendlich ausgedehnt ist oder nur endlich. Beide Möglichkeiten sind gleich spekulativ.

    Wieder falsch. Das Universum hat für uns eine prinzipbedingte Grenze. Und diese Grenze hängt von der Entfernung zu uns ab. Hatte ich aber bereits erwähnt.

    Unendlichkeit ist also (leider) genau so möglich wie Endlichkeit.

    Jetzt muß ich lachen. Du hast keine Ahnung von nix, aber erdreistest dich, das Universum und alle seine Gesetze so gut verstehen zu glauben, daß du aus der hohlen Hand eine 50-50-Wahrscheinlichkeit zauberst und die dann auch noch für realistisch hältst? Mach dich lächerlich. Genausogut könntest du behaupten, daß eine Münze, die du aus 10 m Höhe auf eine glatte Fläche wirfst, entweder auf der Kante stehenbleibt oder eben nicht. Daraus ergäbe sich also eine 50/50-Chance, daß sie auf der Kante steht. Was ich davon halte, muß ich nicht erwähnen, oder?

  195. #196 Auto-Cogitator
    29. Juli 2016

    Langsam wird es unappetitlich mit persönlich werdenden Anwürfen.

    Ebenfalls Wikipedia: “Die derzeit populärste (autotrophe) Theorie zur Entstehung des Lebens postuliert die Entwicklung eines primitiven Stoffwechsels auf Eisen-Schwefel-Oberflächen unter reduzierenden Bedingungen, wie sie im Umfeld vulkanischer Ausdünstungen anzutreffen sind.[4] Während dieser Phase der Evolution auf der Erde, die im Äon, vor zwischen 4,6 und 3,5 Milliarden Jahren, stattfand, war die Erdatmosphäre wahrscheinlich reich an Gasen, vor allem Kohlenstoffdioxid, Wasserstoff und Kohlenstoffmonoxid”

    Die Datierung der ältesten fossilen “eindeutigen Lebensspuren” als den Beginn der (biologischen) Evolution zu setzen ist ja so intelligent. Denn natürlich wird das erste Leben peinlich darauf bedacht gewesen sein, ordnungsgemäß zu versteinern sowie bis spätestens 2016 gefunden zu werden.

    Zur Unwahrscheinlichkeit der Entstehung intelligenten Lebens tragen übrigens die Unwägbarkeiten der kosmischen und der chemischen Evolution vor der biologischen ebenso bei, deshalb kann man auch von 13,8 Milliarden Jahren Gesamtevolution sprechen. (Spätestens mit der Entwicklung der ersten menschenähnlichen KI wird man von der technischen Evolution als der nächsten Stufe nach der biologischen und kulturellen Evolution sprechen müssen.)

    Du siehst tatsächlich nicht den fundamentalen Unterschied zwischen deiner albernen Würfelei mit einem gezielten Aussortieren der Sechser und der völlig ohne vorgefassten Plan ablaufenden biologischen Evolution, aber maßt dir an, mir Intelligenz abzusprechen?

    Es gibt schlicht keinerlei Daten, anhand derer man entscheiden könnte, ob das Universum endlich oder unendlich ist, daraus ergibt sich automatisch die Fifty-fifty-Wahrscheinlichkeit. Bei dem Münzwurf ist es völlig anders, da es jede Menge Daten und Erfahrungen gibt, um sagen zu können, dass ein Landen auf der Kante weniger wahrscheinlich ist. Wenn dir Daten in Bezug auf die Ausdehnung des Universums bekannt sind, die eine Variante wahrscheinlicher erscheinen lassen, dann her damit. Hast du das verlinkte Video von Florian Freistetter zu dem Thema angesehen? Oder hat er für dich auch “keine Ahnung von nix”? (Das ist zwar eine doppelte Negation und somit synonym mit “Ahnung von allem”, aber ich unterstelle mal, unser erklärter Intelligenzexperte versuchte hier eine Beleidigung.)

    Auf diverse wolkig-argumentfreie Arroganzausbrüche in dem Beitrag gehe ich mangels Satisfaktionsfähigkeit nicht näher ein.

  196. #197 PDP10
    29. Juli 2016

    @Auto-Cogitator:

    Es gibt schlicht keinerlei Daten, anhand derer man entscheiden könnte, ob das Universum endlich oder unendlich ist, daraus ergibt sich automatisch die Fifty-fifty-Wahrscheinlichkeit.

    Nein. Du sagst ja selber, dass es keinerlei Daten gibt welches Szenario das richtige ist. Daher gibt es auch keine Möglichkeit über die Wahrscheinlichkeit des einen oder anderen eine Aussage zu machen.
    Zu behaupten die Wahrscheinlichkeit wäre deshalb Fifty-Fifty weil es genau zwei Möglichkeiten gibt ist daher schlicht Unsinn.

    Das wäre so, wie wenn dich jemand bittet, die Wahrscheinlichkeit auszurechnen, dass ein Würfel mit verschieden schweren Seiten auf eine bestimmte Seite fällt und dir aber nicht sagt, wie schwer die Seiten sind.
    Wenn der unausgewogene Würfel 6 Seiten hat, heisst das in diesem Beispiel noch lange nicht, dass die Wahrscheinlichkeit für eine Seite 1/6tel wäre.

    BTW: Dein arroganter Tonfall geht hier noch mehr Leuten auf die Nerven. Freundlich bleiben. Dann bleiben alle hier freundlich. Ganz einfach.

  197. #198 Bullet
    30. Juli 2016

    Es gibt schlicht keinerlei Daten, anhand derer man entscheiden könnte, ob das Universum endlich oder unendlich ist, daraus ergibt sich automatisch die Fifty-fifty-Wahrscheinlichkeit. Bei dem Münzwurf ist es völlig anders, da es jede Menge Daten und Erfahrungen gibt, um sagen zu können, dass ein Landen auf der Kante weniger wahrscheinlich ist.

    Aha. Das heißt deiner Meinung nach, daß ein Münzwurf wie der von mir skizzierte solange eine 50/50-Wahrscheinlichkeit hat, das Ergebnis “Kante” zu zeigen, bis eine genügend große Datenbasis dir zeigt, daß dem nicht so ist. Das Universum paßt die Statistik also deiner Erwartungshaltung an. Soso. Widersprüche hiergegen kannst du dir sparen, denn genau das hast du explizit behauptet. Nochmal: du behauptest, daß, wenn wir eben die vielen Daten beim Münzwurf nicht hätten, die Wahrscheinlichkeit dafür, daß eine Münze tatsächlich auf der Kante stehen bleibt, 50% wäre.
    Und dem ist eben nicht so. Man könnte sie aber natürlich fälschlicherweise auf 50% ansetzen, weil das sympathisch klingt. Aber wie schriebst du selbst weiter oben:

    Aber leider schert sich die Natur nicht darum, was irgendjemand sympathisch oder unsympathisch findet.

    Ach, und welche “persönlichen Anwürfe” meinst du? Daß du keine Ahnung von nix hast? Nun, die einzige andere Möglichkeit ist, daß du voll der Wisser bist. Und das bist du nicht. Das ist kein Mensch. Gemessen an dem, was es zu wissen gibt, weißt du nix. q.e.d.
    Oder etwa “du weißt nicht einmal, was Intelligenz ist”? Nun, ich weiß es zumindest nicht. Ich weiß nur, daß alle möglichen Versuche, Intelligenz zu quantifizieren, grandios gescheitert sind (vgl. Lachnummer “IQ”), und daß es scheinbar 38 verschiedene Formen von Intelligenz gibt, die nur in ihren individuellen Kontexten anwendbar sind. Klingt mir nicht nach einer belastbaren Definition von Intelligenz. Aber vielleicht hab ich mich ja geirrt, und du hast da voll den Plan. Ich höre und lausche also ergriffen deiner weisen Erklärung, was denn Intelligenz sei.

    Die Datierung der ältesten fossilen “eindeutigen Lebensspuren” als den Beginn der (biologischen) Evolution zu setzen ist ja so intelligent.

    O ja, ist es, denn alles andere ist Spekulation.

    Unterschied zwischen […] Würfelei […] und der völlig ohne vorgefassten Plan ablaufenden biologischen Evolution […]

    Wenn es keinen vorgefaßten Plan gibt, heißt das nicht, das nicht die ersten Schritte festlegen können, welche Schritte danach noch möglich sind, newar? Und wenn die Möglichkeiten im Lauf der Zeit immer weiter eingeschränkt werden, ist das schon kein völliger Zufall mehr. Das ignorierst du in deiner sog. “Wahscheinlichkeitsbetrachtung” allerdings konsequent.

    […] aber maßt dir an, mir Intelligenz abzusprechen?

    Wo war das nochmal genau?
    Letztes Bonmot:

    Auf diverse wolkig-argumentfreie Arroganzausbrüche in dem Beitrag gehe ich mangels Satisfaktionsfähigkeit nicht näher ein.

    Wenn du wüßtest, wie oft schon Kommentatoren/innen in diesem Blog genau das gesagt haben, wenn sie nicht mehr weiter wußten …

  198. #199 Dampier
    30. Juli 2016

    @Bullet

    Ach, und welche “persönlichen Anwürfe” meinst du?

    Vielleicht deine sehr unhöfliche und unnötig scharfe Art, zu diskutieren?

    Wenn du wüßtest, wie oft schon Kommentatoren/innen in diesem Blog genau das gesagt haben, wenn sie nicht mehr weiter wußten …

    … oder wenn sie einfach keine Lust mehr hatten, mit dir zu diskutieren, aufgrund o.g. schlechten Stils?

    Naja, Stammleser kennen das ja. Ich zumindest merke, dass ich deine Beiträge mittlerweile öfter einfach überlese, weil ich keine Lust auf diese Rumpelstilzchen-Nummer habe (keine Lust, mir bei einem eigentlich interessanten Thema die Laune verderben zu lassen). Was schade ist – es ist ja nicht so, dass du nichts interessantes beizutragen hättest, aber der Ton macht die Musik, und dein Ton ist auch mir oft zu schrill.

    Soweit mein Beitrag zur Metadiskussion, nix für ungut, Gruß
    Dampier

  199. #200 Alderamin
    30. Juli 2016

    @Auto-Cogitator

    Sorry, war unterwegs.

    “Nee, gerade so einfach geht’s nicht, die Schwerkraft der Erde nimmt außerhalb derselben mit 1/r² ab, da müsstest Du mit Newtons Gravitationsgesetz rechnen.”

    Die Vorausssetzung bei der Rechnung war ausdrücklich “nahe der Erde”, also gerade ohne nennenswerten Abstand.

    Wenn schon die Gravitationskraft an der Erdoberfläche angesetzt wird, dann ist “nahe” an der Erde auch schon “verdammt nahe”, dann die Schwerkraft nimmt mit dem Quadrat der Entfernung ab. D.h. in 2 Erdradien Entfernung (6380 km Höhe über dem Erdboden) beträgt sie nur noch 1/4 des Wertes an der Erdoberfläche. Verdammt nahe heißt dann aber auch, dass man da nur im freien Fall hingelangt, mit ordentlicher Endbeschleunigung. Oder man ist mit einer Seitwärtskomponente im Orbit, dann fällt man aber auch nicht auf den Erdboden.

    Es gibt keine Chance, aus den Tiefen des Weltraums mit einer Endgeschwindigkeit von 0 in die Nähe der Erde zu gelangen (es sei denn, man hätte einen Antrieb). Man rollt immer irgendwie in einen tiefen Potenzialtrichter hinein.

    “Aber solange die Kräfte bei An- und Abflug symmetrisch sind, kommt in beiden Fällen heraus, dass das Objekt wieder auf die gleiche Entfernung kommen wird.”

    Völlig symmetrisch sind sie nur bei Anfangsgeschwindigkeit 0. Und schon kleinste Reibungsverluste führen zu Asymmetrie und zur Annäherung and den Erdkern. Wie gezeigt sogar bei Geschwindikeiten über 0, da ab dem Eintauchen in die Erde die Schwerebeschleunigung bis zum Mittelpunkt immer schwächer wird, was bei der Rechnung nicht berücksichtigt wurde.

    Symmetrisch in dem Sinne, dass man am Ende die ursprüngliche Geschwindigkeit (dem Betrage nach!) relativ zur Erde hat. Wenn man als im Abstand r mit v daher kam, dann wird man im Abstand r nach dem Vorbeiflug wieder v haben, nur ggf. eine andere Richtung (wird bei Swing-By-Manövern genutzt).

    Und was Reibung betrifft: was soll am Ereignishorizont eines schwarzen Lochs von ca. 1/10 Mondmasse mit rund 1 mm² Querschnitt an Reibungsverlusten auftreten? Das ist das perfekte Geschoss!

    Nun zur dann brisant werdenden Frage 2.

    Ich schrieb:

    “Praktisch gesehen dürfte doch die Anziehungskraft der Erde auf weit (gar unendlich weit) entfernte Objekte angesichts der übrigen Massen im Universum in allen Richtungen vernachlässigbar sein.”

    Und:

    “st es denkbar (wenn auch nicht wahrscheinlich), dass ein primordiales Schwarzes Loch eine solch geringe Geschwindigkeit (relativ zur Erde) hat? (Vielleicht, weil ein Galaxienhaufen in entgegengesetzter Richtung zur Erde auf das PSL auf dessen Weg durchs Universum einen relativ zur Erde abbremsenden Einfluss hatte?)”

    In deiner Schilderung gibt es nach wie vor die Erde, die auf das PSL schon in unendlicher Entfernung einen maßgeblichen Einfluss gehabt habe, das überzeugt mich nach wie vor nicht. Auch auf mögliche abbremsende Schwerkrafteinflüsse durch andere Massen bist du nicht eingegangen.

    Die spielen in der Erdnähe keine Rolle mehr. Sagen wir mal, in Mondentfernung (ca. 400.000 km) hätte das Objekt irgendwie eine Ausgangsgeschwindigkeit von 0, das ist schon tief innerhalb der Hillspäre. Die potenzielle Energie im Schwerefeld beträgt Epot = -G m*M/r.

    r1 sei die Entfernung beim Mond, r0 die an der Erdoberfläche (ca. 6400 km vom Erdzentrum entfernt), dann ist r1 = 62,5 * r0.

    Sei die Geschwindigkeit im Abstand r v(‌r), dann ist die kinetische Energie dort 1/2 m v²(r‌). Überall im freien Fall gilt, dass die Summe aus kinetischer und potenzieller Energie konstant sind: Ekin(r‌) + Epot(r‌) = 1/2 mv²(r‌) – GmM/r = const. Aufgelöst nach v(r‌) ergibt sich v(r‌) = √2 * √(const./m + GM/r). Setzen wir r1 ein, wo v(r‌) definitionsgemäß =0 sein soll, so gilt v(r1) = 0 = √2 * √(const./m + GM/r1). Dies ist genau dann erfüllt, wenn der Wurzelterm 0 ist, also const./m = -GM/r1. Wir setzen dies in die Gleichung für v(r0) ein: v(r0) = √2 * √(const./m + GM/r0) = √2 * √(-GM/r1 + GM/r0). Wir setzen ein r1 = 62,5*r0: v(r0) = √2 * √(GM/r0-GM/r1 ) = √2 * √(GM/r0-GM/(62,5r0)) = √(2GM/r0) * √(1-1/62,5). Der erste Ausdruck entspricht genau der Fluchtgeschwindigkeit der Erde in Meereshöhe, die bekanntlich 11,2 km/s ist. Ergo ist die Geschwindigkeit eines Objekts, das mit 0 km/h aus der Höhe des Mondes losgelassen wird, auf Meereshöhe (ohne Luftwiderstand) v(r0) = 11,2 km/s * √(1-1/62,5) = 11,11 km/s. Wie man sieht, entspricht dies schon bis auf die erste Nachkommastelle der Geschwindigkeit für einen freien Fall aus dem Unendlichen.

    Also: Selbst wenn ein Objekt in Mondentfernung die Geschwindigkeit 0 hätte, würde es praktisch mit der Fluchtgeschwindigkeit der Erde einschlagen. Und zwischen Erde und Mond ist nichts, was die Geschwindigkeit noch verändern könnte (innerhalb der Hill-Sphäre kann man alle anderen Körper vernachlässigen).

    Gewichtiger scheint mir das Argument, dass schon die Sonne für eine Beschleunigung über die Fluchtgeschwindikeit von der Erde sorge. Du kommst dabei auf eine Mindestgeschwindigkeit von 17 km/s, das wäre in der Tat 5,8 km/s über der Fluchtgeschwindigkeit. So richtig beruhigend weit davon weg wäre das aber nicht.

    Doch, denn der oben gerechnete freie Fall auf die Erde kommt ja dann noch dazu. Die 17 km/s gelten nur auf einer Bahn, die die Erde von hinten einholt. Und dann kommen nicht 5,8 km/s heraus, denn das wäre ja der Fall, wenn man die Fluchtgeschwindigkeit von der Annäherungsgeschwindigkeit subtrahieren würde. Tatsächlich muss man addieren (hier wird aber nicht linear addiert, die Anfangs-Annäherungsgeschwindigkeit verkürzt die Zeit, während der die beschleunigende Kraft wirkt; jedenfalls wird die Geschwindigkeit bis zum Einschlag nicht kleiner als 17 km/s, sondern größer.

    Konkretes Szenario: Das PSL kommt von außern ins Sonnensystem in Richtung auf die Erde. Unterwegs sind die Bahnbewegungen der Planeten, denen es begegnet, gerade so ungünstig, dass deren Schwerkraft im Vorbeiflug auf ds PSL in Richting auf die Erde eine abbremsende Wirkung hat. Das scheint mir doch debnkbar.

    Nein. Denn Abbremsen kann die Erde das Objekt nur, wenn es sich von ihr entfernt (wie etwa ein Stein, den man hochwirft; der wird nur auf dem Weg nach oben langsamer). Dafür muss es sich aber vorher annähern, und bei der Annäherung wird es beschleunigt. Der Anteil der Erde hebt sich im Koordinatensystem der Erde insgesamt auf (gleicher Abstand- gleiche Geschwindigkeit). Ein Einschlag auf der Erde mit 0 km/h ist genau so ausgeschlossen, wie der Einschlag eines Steines auf der Erde mit 0 km/h, den man aus großer Höhe fallen lässt (oder nach oben schmeisst, was auch nicht hilft). Das ist nämlich beides dasselbe.

  200. #201 Krypto
    30. Juli 2016

    @Auto-Cogitator:

    Was ich nicht verstehe: Ein schwarzes Loch von der Masse eines Asteroiden müsste doch von der viel größeren Masse der Erde schwerkraftmäßig eingefangen werden.

    Das wurde Dir dann ausführlich erklärt.
    Dann flüchtest Du Dich immer weiter auf rein theoretische Möglichkeiten mit winzigsten Wahrscheinlichkeiten.

    Dass das Szenario von geringer Wahrscheinlichkeit ist, sei attestiert. Es geht um Denkbarkeit.

    Wozu? Ich habe den Eindruck, hier geht es nur um Rechthaberei und schon lange nicht mehr um´s Thema.

  201. #202 Auto-Cogitator
    31. Juli 2016

    Das Thema (PSL-Kollision zur Lebzeit der Menschheit) handelt ohnehin von einer winzigen Wahrscheinlichkeit – laut Artikel ist nur alle paar Millionen Jahre ein PSL zu erwarten.

    Und erklärt habe ich es mir selber mit den Geschwindigkeitsbetrachtungen.

    Mich wunderte einfach, dass die Möglichkeit des Einfangens eines nur asteroidenschweren Objekts im Schwerefeld der Erde bisher noch nicht thematisiert worden war. Also habe ich das nachgeholt und auch gleich auf die Schlüsselgröße für die Beantwortung dieser Frage hingewiesen, nämlich auf die Fluchtgeschwindigkeit, die vorher auch noch keine Erwähnung gefunden hatte. Ohne diese Betrachtungen erschließt sich gar nicht, warum eine Begegnung der Erde mit einem PSL in den meisten Fällen harmlos wäre. Der Artikel begründet die Harmlosigkeit nur mit der geringen Größe der PSL und mit der Kürze des Erdtransfers, informiert aber nicht über die anschließende Flucht aus dem Schwerefeld.

    Für studierte Physiker mit der Fluchtgeschwindigkeit von der Erde auswendig im Kopf war das vielleicht selbstverständlich, ich nehme aber nicht an, dass sich dieser Blog nur an studierte Physiker richtet.

    Die Möglichkeit einer Abbremsung des PSL unter die Fluchtgeschwindigkeit war dann die absolut naheliegende Anschlussfrage.

    Jetzt ist für mich die Frage naheliegend, warum man auf unwahrscheinliche, aber denkbare Planetenkiller-Szenarien so aggressiv reagiert. Es gibt sie halt. Auch Killerasteroiden, Gamma-Ray-Bursts, Rogue Planets, Neutronensterne, Vakuumzerfall und weiß der Teufel was sonst noch. Hängt nicht die Boten. Und so ein dolles Aushängeschild für eine lohnende Zivilisation ist der Auschwitz-Planet nun auch nicht.

    Den Exkurs zur Unendlichkeit fand auch niemand interessant? Seltsam. Dazu noch ein Versuch, es zu verklickern:

    Wenn es nur zwei Möglichkeiten gibt und man noch keinerlei empirische Anhaltspunkte hat, die eine der beiden Möglichkeiten plausibler erscheinen lassen, dann muss man bis dahin (bei intellektueller Redlichkeit statt Wunschdenken) mit beiden Möglichkeiten gleichermaßen rechnen. Also: Fifty-fifty.

    Natürlich ist das nicht die “objektive Wahrscheinlichkeit”, mit der das Universum endlich oder unendlich ist. Objektiv ist das gar keine Wahrscheinlichkeitsfrage, sondern es ist mit absoluter Sicherheit entweder so oder so – nur gibt es offenbar (wenn ich u.a. das oben verlinkte Video richtig verstanden habe) noch keinerlei Anhaltspunkte, um es auch nur abschätzen zu können.

    Nachdem ich mir den Artikel oben noch mal durchgelesen habe, muss ich noch eine Hiobsbotschaft loswerden (hängt mich höher). Da steht zum Szenario eines stellaren schwarzen Lochs mit Kurs auf das Sonnensystem:

    “Es ist so schwer wie ein großer Stern und wir würden lange vorher merken, dass sich die äußeren Planeten und Asteroiden unseres Sonnensystems nicht mehr so bewegen, wie sie es tun sollten, weil ihre Bahn vom nahenden schwarzen Loch gestört werden. Um die stellaren schwarzen Löcher müssen wir uns also keine Sorgen machen.”

    Warum soll man sich keine Sorgen machen? Weil man von dem sicheren Ende aufgrund der zu erwartenden Änderung der Erdbahn “lange vorher” wüsste? – Es ist ja nicht so wie bei einem rechtzeitig entdeckten Asteroiden auf Erdkurs, gegen den realistische Abwehrmaßnahmen denkbar wären. Der einzige Ausweg, der mir einfiele, wäre eine Evakuierung der Menschheit aus dem Sonnensystems mittels interstellarer Raumfahrt. Aber ob wir so viel Zeit hätten? Und wenn wir bis dahin noch nicht mal das Geld überwunden haben: Wer soll das bezahlen? Außerdem hat man bestimmt nicht mehr genug seltene Erden für so viele Raumschiffe. Denkt darüber mal nach, bevor Ihr Euch das neueste iphone holt, obwohl es das alte noch tut.

  202. #203 Auto-Cogitator
    31. Juli 2016

    @ Alderamin: Danke für deinen sachlichen und interessanten Beitrag, und insbesondere danke für die Mühe bei der Rechnung. Bis zur Absetzung meines letzten Beitrags hatte ich deinen Beitrag irgendwie übersehen, deshalb ging er darauf noch nicht ein. Sobald ich die Zeit finde, werde ich das eine Weile auswerten und darüber nachdenken, bevor ich dazu Stellung nehme.

    (Hoffentlich überschneidet sich das nicht mit anderen Beiträgen. Ich bekomme immer die Meldung, dass meine Beiträge erst moderiert werden, also wohl zeitverzögert erscheinen. Für Normalsterbliche sehe ich hier keine Möglichkeit, sich zu registrieren, auch für einen besseren Editor mit Zitatfunktion usw. Oder geht das irgendwie?)

  203. #204 Alderamin
    31. Juli 2016

    @Auto-Cogitator

    Die Mod‌eration erfolgt, wenn bestimmte Stich‌worte im Text enthalten sind, die auf S‌pam hindeuten. Manchmal werden Artikel auch schon eine Stufe früher block‌iert, die erscheinen da nicht mal als mo‌deriert, sondern gehen gleich in die Müll‌tonne. Es gibt keine Reg‌istrierung, die einen an diesen Filtern vorbei schleust. Man muss einfach Geduld haben, bis Florian mode‌rierte Beiträge (wie meinen oben) freischaltet, oder ihn bitten, nicht angezeigte Beiträge aus dem S‌pamordner zu holen. Da er auch mal Urlaub macht, kann das schon mal einen Tag dauern.

  204. #205 Auto-Cogitator
    19. August 2016

    Danke für die Infos. Gibt es eine Liste der bösen Wörter? Und wie kommt man an die Zitatfunktion? Kann man irgendwelche Tags nutzen? (BB-Code scheint nicht zu gehen.)

    Deine Rechnung kann ich leider im Detail nicht nachvollziehen, bin aber keine mathematische Leuchte, also könnte sie stimmen. 🙂 Da ich auch in mehreren Anläufen nicht geschafft habe, einen anderen Rechenweg zu ergoogeln, kommt diese Antwort spät, sorry.

    Wenn das Rechenergebnis stimmt, hast Du damit alle offenen Fragen geklärt und den Planetenkiller gekillt. Du kommst auf 11,11 km/s auf Meereshöhe, die weitere Beschleunigung bis zum Erdkern wird dann in der Tat ein Erreichen der Fluchtgeschwindigkeit garantieren. Das bei einem Ausgangswert von v = 0 im Mondabstand ohne andere abbremsende Massen in Sicht – das wäre überzeugend.

    Dass die Reibung eines PSL mit der Erdmaterie während eines einmaligen kurzen Transits verschwindend gering wäre, war mir schon klar, aber schwenkte ein (theoretisches) PSL-unter-Fluchtgeschwindigkeit in einen Orbit um den Erdkern ein, so hätte es von da an ja alle Zeit der Welt, seine Geschwindigkeit abzubauen; irgendwann müsste auch diese minimale Reibung dazu führen, dass der Orbit enger wird.

    Um besser zu verstehen, wie die Beschleunigung im Erdkern verläuft, stieß ich beim Googeln auf Newtons Schalentheorem: Wenn in einer Hohlkugel von Erdschwere sämtliche Masse in einer dünnen (dann freilich unrealistisch stabilen) Schale bestünde, würde man darauf stehend sein Gewicht normal mit 1 g spüren; öffnet man eine Luke und lässt sich herab, so kann man den Rand der Öffnung loslassen und würde schweben – im gesamten inneren Hohlraum würden sich die Anziehungskräfte der Schale auf null ausgleichen – verrückt. (Aufgrund des Schalentheorems bräuchte man bei einem Fall ins Innere der Erde nur die abnehmende, noch vor dem fallenden Objekt liegende Masse berücksichtigen, die anwachsende schalenförmige äußere Masse hebt sich in ihrer Schwerkraftwirkung auf.)

  205. #206 Max
    19. Februar 2017

    @Aventin #134 usw
    Ich habe mir schon vor Jahren, bei allen Überlegungen zunächst die Zeit herausgenommen. (speziell bei den Schwarzen Löcher) ..[weil sie ja variabel ist].
    Daher also auch keine direkte Geschwindigkeit Strecke/Zeit, nur die Interaktion und Abhängigkeit beachtet. …es hilft^^ , man muß nicht alles 1:1 glauben.
    Deinen Gedankenweg bin ich auch gegangen 🙂

    Hier ein Link zu Fermi Gamma-ray Space Telescope.

    https://www.nasa.gov/mission_pages/GLAST/news/new-structure.html

    mit den zwei Sphären, ich finde dies gehört einfach dazu.

  206. #207 Max
    20. Februar 2017

    Nachtrag:
    “Neben den großen stellaren schwarzen Löchern, die aus Sternen entstehen, gibt es aber vielleicht auch noch eine andere Art. Winzig kleine, sogenannte “primordiale” schwarze Löcher.”

    Es gibt keine PSL, …das was LHC aus Protonen-paaren untersucht, soll zerfallenen und Informationen liefern, ein PSL “primordiale schwarze Loch” in der Größenordnung eines Asteroiden, wäre eben nicht SCHWARZ ! auch wenn die Materie extrem verdichtet ist. Kompakte Massen? zB. Magnetare (wäre als Beitrag interessant^^)
    Genau so gut, könnte man sich einen (BRS) Bösen Rosa Schleim ausdenken, der sich tarnen kann, und gelegentlich noch Dunkle Energie ausstößt. 😉

  207. #208 Mimo
    Berlin
    13. August 2020

    Dann müsste ja irgendwo ein kleines Loch durch die Erde gehen, wenn es sich bei der Konfrontation mit einem schwarzes Loch, wie bei einer Patrone verhält, die durch eine weiche Melone durchschießt….?

  208. #209 Karl-Heinz
    13. August 2020

    @Mimo

    Und was denkst du, was so ein primordiales schwarze Loch für einen Durchmesser hat?
    Was würdest du dazu sagen, wenn ein primordiale schwarze Loch kleiner als ein Proton ist? Also nix mit Kaliber neun. 🙂